Thread Rating:
  • 0 Vote(s) - 0 Average
  • 1
  • 2
  • 3
  • 4
  • 5
Conrad Fischer's lectures on Internal Medicine - mira1978
#31
bump
Reply
#32
bump..to the top.
Thank u mira. Wonderful work
Reply
#33
http://egymedicine.com/forums/thread2784.html

(different way to download)
Reply
#34
my second time bump
Reply
#35
1. A 25- year old mother refused immunization for her 2-month old son. The social worker spoke to the mother. (Important for Board examination)
Next step in management: immunization should be given for the benefit of the child.
2. A 30-year old mother refused surgery for suspected appendicitis for her 6-year old daughter. The social worker spoke to the mother. (Important for Board examination)
Next step in management: surgical removal of the appendix should be preformed for the benefit of the child.
3. A 16-year old boy was diagnosed with osteosarcoma of the right thigh. The surgeon recommended amputation. The boy refuses amputation. He is doing very well otherwise. He is aware that death is certain without surgery. (Important)
Next step in management: amputation should NOT be preformed.
Adolescent patients or adults who are competent in making decisions have an absolute right to determine what shall be done with their own bodies. However, most pediatric patients are not competent to make their own decisions. Please remember, children (15 years or older) are usually able to give a genuinely informed consent. Therefore physicians may respond to their request, except in a case of irreversible sterilization.
4. A 17-year old girl is a Jehovah's Witness. She refuses a lifesaving blood transfusion. She is aware of the consequences. She spoke to the social worker. (Important)
Next step in management: blood transfusion should NOT be given because she is competent to make the decision.
5. A 5-year old girl is a Jehovah's Witness. She requires emergency blood transfusion. Her mother refuses blood transfusion. A social worker along with two physicians spoke to the mother. (Important)
Next step in management: blood transfusion should be given because the patient is not competent. Mother cannot refuse her daughter's treatment.
6. A 2-year old boy was brought to the ER by his parents for an injury. Physicians made the diagnosis of child abuse. There are three other children living in the same household. Both parents confess to child abuse but request the physician to keep it confidential. Social worker was involved. (Important)
Next step in management: the case should be reported to Child Welfare Agency (CWA). All children should be removed from the parents.
7. A 15-year old boy with STD (sexually transmitted disease) came to see a physician. He asked the physician not to tell his parents. (VERY IMPORTANT)
Next step in management: the physician should treat the patient and notify the appropriate health authority, but should not tell his parents.
8. A 16-year old boy wants to use a condom. He comes to the clinic for free samples. He requested the physician, however, not to tell his parents. (Important)
Next step in management: condom should be given and physician should not tell his parents.
9. A 30-year old male patient is recently diagnosed with HIV. He lives with his wife and two other children but is promiscuous. He requested the physician not to tell his wife. He lost his job recently. Social worker spoke to the patient. (Very Important)
Next step in management: physician should notify the appropriate authority (e.g. department of health) for the safety of other specific persons who are engaged in unsafe sexual practices. The physician however, should ask the patient to divulge the diagnosis to his wife and other sexual partners.
10. A very small premature infant was born in the delivery room by NSVD (normal spontaneous vaginal delivery). The attending physician decided not to resuscitate the newborn. Physician spoke to the mother. Mother started to cry. Newborn expired after 30 minutes. Is the physician liable for the newborn's death?
Answer: NO. Please remember, no physician in the USA has ever been found liable for withholding or withdrawing any life sustaining treatment from any patient for any reason.
11. A physician picked up a car accident victim from the street and brought him to the ER in his car. He did not want to wait for an ambulance because the patient's condition was critical. Physical examination in the ER reveals quadriplegia. Is the physician liable for this consequence? (Very Important)
Answer: YES, because the physician did not protect the neck of the patient resulting in quadriplegia.
12. A policeman brought an alcoholic patient to the ER. The policeman asked the physician to give him a sample of gastric contents by putting a nasogastric tube for laboratory study. The patient refused insertion of a nasogastric tube. (Very Important)
Next step in management: nasogastric tube should not be placed. Blood alcohol level however should be preformed. The policeman should not give orders to a physician.
13. A 60-year old man with a history of myocardial infarction (MI) suddenly develops ventricular tachycardia. A physician from another department was present. The patient needs resuscitation. (Very Important)
Next step in management: the physician must resuscitate that patient. Physician should not refuse treatment because he belongs to another department.
14. A 15-year old homosexual boy wanted to change his sexual orientation. He was not successful. He needed help. He requested the physician not to tell his parents. (Important)
Next step in management: physician should help him avoid homosexual activities. Physician should not tell his parents about his homosexual activities.
15. A 15-year old homosexual boy is brought by his parents to a physician. His parents do not accept their son's sexual orientation. The boy refuses to change. (Important)
Next step in management: physician should tell his parent that homosexual activity is considered as an alternative life style. Parents should talk to his son but should not force him to change his homosexual activities.
16. A 16-year old girl becomes pregnant. Her mother wanted her to abort this pregnancy. The girl wanted to continue her pregnancy despite difficulties. Her boyfriend is a high school drop out. (Important)
Next step in management: physician should advise to continue this pregnancy because the girl is competent to make this decision.
17. A 15-year old girl recently becomes pregnant. She went to a doctor for abortion. She told the doctor not to tell her parents about this pregnancy. (Important)
Next step in management: abortion should be done and parents should not be notified. Please remember, strict requirements for parental consent may deter many adolescents from seeking health care.
18. Can a physician provide sterile needles for intravenous drug abusers? (Important)
Answer: YES. It reduces the risk of acquiring HIV or hepatitis. The patients should be referred to appropriate health facilities.
19. A 26-year old pregnant woman went for antenatal check up. Sonogram revealed a 27-week old fetus with erythroblastosis fetalis. Doctor recommended intrauterine fetal blood transfusion. She refused the procedure. Social worker discussed the case with the mother. (Important)
Next step in management: doctor should receive a court to do the procedure which will help the fetal condition.
20. A 20-year old man tells his doctor that he is going to kill girlfriend. She lives in the university dormitory. Doctor called the university and alerted them about the threat. However, university security people did not protect her. She was killed by her boyfriend. Who should be responsible for this killing? (Important)
Answer: the 20-year old man is responsible. Doctor did the right thing by notifying the university or the police. The university is also responsible because they did not take any preventive measures.
21. A 20-year old pregnant woman refuses cesarean section for complete placenta previa. Fetus is full-term and healthy. Social worker spoke to mother. (Important)
Next step in management: doctor can go to court to get permission for cesarean section for the benefit of the fetus.
22. A 30-year old pregnant woman ingested alcohol and illicit drugs (e.g., cocaine, crack) which are harmful to the fetus. What should a physician do? (Important)
Answer: the physician should be careful in reporting this case because the pregnant woman may not come back for prenatal care, which is important for both the mother and the fetus. However, if the baby's urine toxicology test is positive for illicit drugs, case should be reported to CWA (child welfare agency). CWA suggests separate custody for the child.
23. A physician wants to study a group of children aging from 10-12 year old. Physician already got the consent from the parents. However, he didn't discuss the study with the children. A child refused to participate. Should the physician force the child to participate? (Important)
Answer: no, because a child can refuse to participate in a research study.
24. A 40-year old schizophrenic patient needs hernia repair. Surgeon discussed the procedure with the patient who understood the procedure. Can the patient give consent? (Important)
Answer: yes. If a psychiatric patient understands the procedure, he or she can give the consent.
25. A 65-year old schizophrenic patient needs coronary angiography because of suspected myocardial infarction. Cardiologists explained the procedure to the patient who did not understand the procedure. Who can give the consent on behalf of the patient? (Important)
Answer: the patient's relative can give the consent. If nobody is available to give the consent, court order should be obtained. If a psychiatric patient does not understand the procedure, he or she cannot give the consent.
26. A 25-year old woman developed postpartum psychosis. The newborn developed cyanosis due to congenital heart disease. The newborn needs cardiac surgery. Surgeon discussed the procedure with the mother. She understood the procedure. Can she give the consent? (Important)
Answer: yes, because she understood the procedure.
27. A newborn is diagnosed with either trisomy 18 or 13 with TE (tracheoesophageal) fistula which requires suregery. Mother request surgeon to repair the TE-fistula. What should a surgeon do? (Important)
Answer: surgeon should refuse to do the reparative surgery because these conditions (trisomy 18 or 13) are nonviable. If the patient survives, surgeon can put a gastrostomy feeding tube for nutrition. However, please remember that a patient with trisomy 21 (Down syndrome) with TE fistula should be operated on.
28. A 45-year old terminally ill patient wanted to die. He has pancreatic cancer and has been suffering from constant pain. He asked the physician to give him some medication which can expedite his death. What should a physician do? (Very Important)
Answer: physician cannot give any medication which will expedite the death. However, physician can prescribe medication to minimize the pain. The dose should be appropriate. Physician-assisted suicide is illegal everywhere (except in the state of Oregon).
29. A 47-year old man came to a doctor for chronic low back pain and dysuria. The diagnosis of metastatic prostate cancer was made after appropriate investigation. Should the doctor tell the bad news to the patient? (Important)
Answer: yes physician must tell the truth to the patient.
30. A surgeon wanted to perform cholecystectomy on a patient. The surgeon is not sure whether the patient has decision-making capacity. What is the next appropriate step? (Important)
Answer: consultation with a psychiatrist or neurologist may be helpful. Sometimes it is necessary to discuss the case with hospital attorneys, ethic committees, or ethic consultants. In a difficult case, the ultimate judge of a patient's competency is a court.
31. A 45-year old widow was admitted to an ICU (intensive care unit) with ruptured intracranial aneurysm. She is comatose and is placed on a mechanical ventilator. She has a 20-year old son who did not keep any relation with his mother. However, he came to see his mother. His mother made a written proxy advance directive which indicates that her 50-year old female neighbor should make the substitute decision. Who is the right person to make the substitute decision in this situation? (Important)
Answer: 50-year old neighbor should make the substitute decision. Please remember, the most appropriate person to make the substitute decision is someone designated by the patient while still competent, either orally or through a written proxy advance directive. Other substitute decision makers, in their usual order of priority, include a spouse, adult child, parent, brother or sister, relative, or concerned friend. For a patient who has no other decision maker available, a public official may serve as a decision maker.
32. The right of patients to refuse medical intervention: patients can refuse dialysis, cardiopulmonary resuscitation, mechanical ventilation, and artificial nutrition and hydration, even if such a decision results in the patient's death. A patient's decision to withdraw (discontinue) or to withheld (not to initiate) life-sustaining treatment is not considered suicide and physician participation is not considered physician-assisted suicide. Physicians do not have any legal risk.
33. Can a medical student introduce himself or herself as a 'doctor' to the patient? (Important)
Answer: no. a patient can refuse a medical student from performing any procedure. However, medical students are allowed to perform a procedure under appropriate supervision If the patient agrees to that.
34. Should a bus driver hide history of epilepsy from his employer? (Important)
Answer: no. He has requested his physician not to mention his epilepsy to the employer because this would result in the loss of his job. The physician is obligated not only to his patient but to the community. The patient should notify his employer and try to find a non-driving job in the company. If the patient disagrees, physician may notify the appropriate authority for the safety of the patient and the community.
35. A 50-year old make is diagnosed with stomach cancer. He requested the physician not to tell his wife. The following day, the wife calls to inquire about her husband's diagnosis. (Important)
Answer: the physician should not divulge the husband's diagnosis. However, the physician should encourage the patient to reveal his diagnosis to his wife.
36. A 29-year old man is diagnosed with presymptomatic Huntington's disease. This disease is an autosomal dominant (50% chance of having the disease in each pregnancy). He requested his physician not to tell the diagnosis to his wife. The wife wants to have children. (Important)
Answer: physician should ask the patient to seek genetic counseling and to urge him to discuss the matter with his wife. Since there is a risk of harm to the future children, physician can divulge the diagnosis to protect the future children.
37. A 18-year old man is diagnosed to have suspected bacterial meningitis. He refuses therapy and returns to the college dormitory. What should a physician do in this situation? (Very Important)
Answer: physician should report to the college authority and recommend that the suspected individual should be isolated during the course of his illness.
38. A 39-year old nurse is diagnosed with hepatitis B antigen-positive. She is working in a dialysis unit. She told her doctor. However, she did not tell the hospital authority because she is afraid to lose her job. (Very Important)
Answer: physician should ask the nurse to divulge her medical condition to the hospital authority. If she refuses, physician should notify the hospital authority for the protection of patients.
39. A 20-year old man with severe head injury was admitted to a small hospital. The patient needs neurosurgical intervention which is available in a nearby university hospital. Hospital refused to accept a patient who has no medical insurance.
Answer: university hospital must accept the patient.
40. A 30-year old man needs a second prosthetic valve. He is a drug addict. Surgeon does not want to perform surgery because the patient does not take care of himself. Is this the right decision? (Important)
Answer: no. Surgery should be performed if it is medically indicated.
41. A newborn male is diagnosed with anencephaly. His 1-year old sibling needs a kidney. His parents requested the physician to remove the kidney from the anencephalic child and to transplant that kidney in the 1-year old sibling. What should a physician do?
Answer: surgeon should perform the kidney transplant.
42. A 50-year old man is in a persistent vegetative state. Physician decided to discontinue nutrition and hydration for that patient. Is this the right decision?
Answer: yes. This is an acceptable practice in most states. Few states require clear evidence that the patient would have chosen this course.
43. A medical student requested his attending to perform a pelvic examination on a patient who is anaesthetized for appendectomy. Is this ethically acceptable?
Answer: no. The patient did not give consent to perform a pelvic examination.
44. A 20-year old woman slashed her wrists and wanted to die. She was unconscious and was brought to the ER. What should a physician do? (Important)
Answer: physician should take care of the patient. Psychiatric consultation and social worker evaluation are indicated. A suicide attempt is very often a 'cry for help'.
45. A 90-year old man was diagnosed with having Alzheimer's disease 10 years ago. It is difficult to feed him. He cannot recognize his family members. He developed recurrent aspiration pneumonia. What should a physician do?
Answer: physician should discuss this with the family and should respect their decision.
46. A 1-day-old infant was diagnosed with hypoplastic left heart syndrome. The patient is stabilized with the use of prostaglandin. Physician discussed this case in detail with the parents. What should the parents decide in this situation?
Answer: the parents can choose a staged surgical repair of the heart, a final heart transplantation if the organ is available, or allow the infant to die.
47. A 55-year old woman with severe developmental disability recently is diagnosed with breast cancer. Her mental age is estimated at a 2-year old level. Her family members do not want any more intervention. What should physician do?
Answer: physician should discuss this case with the hospital ethics committee members. The usual consensus is 'not to do anything' because of her severe mental disability.
48. A 49-year old woman with cervical cancer has a history of noncompliance. She had surgery a month ago. She missed several appointments. Can a physician force her for chemotherapy? (Important)
Answer: no. Physician can talk to her regarding the importance of chemotherapy. However, the patient must make the final decision.
49. An internist has been managing a diabetic patient for the last 10 years. The patient's condition is progressively getting worse. The patient is also not happy with the physician's management. What should a physician do in this situation?
Answer: physician should find another physician (e.g., endocrinologist) who might be more successful with the patient in this particular circumstance.
50. An internist recently refused to see a patient who he has been seeing for the last 5 years. Internist stated that the patient was rude to him. The patient went to see another physician who requested the patient's medical record. What should the internist do in this situation?
Answer: internist should provide the medical records of the patient to the new physician.
51. An internist refused to see a complicated hypertensive patient who he has seen for the last 10 years. Internist did not give any notice to that patient. The patient was angry with the physician. The patient was recently admitted to a hospital with the diagnosis of stroke. Is the internist responsible for the patient's condition?
Answer: yes. The legal charge of abandonment can arise when the physician without giving timely notice, ceases to provide care for a patient who is still in need of medical attention. Internist is not obligated to find him another physician. However, patient should have sufficient time to arrange for another physician.
52. A physician went to vacation for 2 weeks. He did not find another physician to cover him. He is very sincere. One of his patients with hypertension developed severe headache. The patient has an appointment with the doctor as soon as he comes back from vacation. The patient did not look for another physician and decided to wait. The patient suddenly collapses and was diagnosed to have intracranial hemorrhage. Is the physician responsible for this patient? (Important)
Answer: yes. The physician has a legal obligation to arrange for coverage by another physician.
53. An ophthalmologist performed a cataract surgery on a patient who went home after the operation. In the evening, the patient started vomiting and complained of severe headache. The ophthalmologist refused to accept that the symptoms were due to postoperative complications. The patient wanted to see the doctor immediately but he refused to see that patient. The patient went to the nearest ER and was diagnosed to have dislocation of the lens and partial retinal detachment. Is the physician responsible for the patient's condition?
Answer: yes ophthalmologist failed to judge the patient's condition seriously enough to warrant attention.
54. A 70-year old Chinese man is diagnosed to have severe osteoarthritis. He told his doctor that he is using Chinese herbal medicine. He is feeling better. However, he had two episodes of dizzy spells since he started that herbal product. What should a doctor suggest to this patient? (Important)
Answer: the doctor should suggest to discontinue the herbal product which may be causing the dizzy spells.
55. A 35-year old woman is diagnosed to have chronic throat infection. She is frustrated with the conventional medicine. She told her doctor that she is using an alternative homeopathic medicine. She is feeling much better and she has no other complications. What should a doctor suggest to this patient?
Answer: the patient can continue an alternative homeopathic medicine. Alternative medicine therapy is accepted in the society and is also used along with conventional therapy.
56. A 45 year old woman is diagnosed to have UTI (urinary tract infection). She told her doctor that she could not afford to purchase antibiotics. However, she is using herbal medicine that is cheaper. She is complaining of fever and dysuria. What should a doctor suggest to this patient? (Important)
Answer: the patient should discontinue the herbal medicine immediately and should start antibiotics as soon as possible.
57. A 13-year old boy with suspected meningitis refuses therapy. His parents also support that decision because they are supposed to go on vacation the following day. What should a physician do in this situation? (Important)
Answer: the patient should be admitted and treated in the hospital. If they refuse, legal action should be taken.
58. A 2-year old girl is admitted with the diagnosis of intestinal obstruction. Her mother has a psychiatric problem. Her mother is not capable of giving the consent. Her father died one year ago. What should a surgeon do in this situation?
Answer: legal steps may be taken to provide a surrogate decision-maker.
59. A 67-year old widow has been using hypnotics for the last 5 years. She is addicted. Her doctor wants to withdraw her from her present medication by trial on placebos. Is the physician making a right decision? (Important)
Answer: no. The physician cannot use placebos because his decision is deceptive. The problem of addiction should be discussed directly with the patient. The use of deceptive placebo is indicated in the following conditions:
(a) the patient insists on a prescription;
(b) the patient wishes to be treated;
© the alternative to placebo is either continue illness or the use of a drug with know toxicity;
(d) high response rates to placebo ( e.g., postoperative pain, mild mental depression).
60. A 50-year old man is diagnosed to have multiple sclerosis. In the morning, the surgeon asked the man his opinion on the surgical procedure and he agreed. In the evening, the man refused to give consent for the same surgical procedure. He is also disoriented to place and time. Is the patient capable of making the decision?
Answer: no the patient has impaired capacity.
61. A 55-year old woman with diabetes is diagnosed to have gangrene on both feet. She was brought to the hospital. She told the doctor the she is feeling fine and she has no medical problems. Can she give consent for the amputation of both legs?
Answer: no. The appointment of a surrogate should be sought to get the consent for the surgery.
62. A 17-year old boy came to a surgeon for bilateral vasectomy. He is the father of one child and does not want to have any more children. He does not want to tell his girlfriend and parents. He lives with his parents. What should a surgeon do in this situation? (Important)
Answer: surgeon should not perform bilateral vasectomy and should offer him less radical alternatives. Please remember, a mature minor may not comprehend the implications of this procedure.
63. A 16-year old girl came to a doctor for bilateral tubal ligations. She is a mother of one child and does not want to have any more children. She does not want to tell her boyfriend and parents. She lives with her parents. What should a doctor do in this situation? (Important)
Answer: obgyn doctor should not perform bilateral tubal ligation and should offer her less radical alternatives.
64. A 16-year old boy wants to donate one of his kidneys to his friend who is suffering from ESRD (end stage renal disease). The boy's parents did not agree with his decision. What should a physician do in this situation? (Important)
Answer: the physician cannot accept his kidney. However, he can donate one of his kidneys if his parents agree.
65. A 15-year old boy wants to participate in a research study. He told his parents who did not agree. He lives with his parents. Can this boy participate in the research study?
Answer: no the boy needs consent from his parents to participate in a research study.
66. A 17-year old boy lives independently. He is married and has one child. He wants to participate in a research study. Does he need his parents permission? (Important)
Answer: no. He is an emancipated minor who lives independently from his parents physically and financially.
67. A 70-year old man is diagnosed with terminal esophageal cancer and requires an insertion of a gastrostomy tube. He has signed a DNR (Do Not Resuscitate) order about a month ago. Should the preexisting DNR order stand or be suspended during the surgical procedure? (Very Important)
Answer: attending physician, surgeons, and the patient or surrogate should discuss the matter and either affirm or suspend the order in anticipation of surgery. If a patient is competent and wishes a preexisting DNR order to stand, resuscitation should not be performed in the event of an intrasurgical arrest.
68. An infant, born at 30 weeks gestation, appears to be SGA (small for gestational age) with multiple malformations. Amniocentesis study was not performed. Infant needs resuscitation at birth. What should a physician do in this situation? (Important)
Answer: physician must resuscitate the patient in the delivery room because the diagnosis is uncertain.
69. A 60-year old man is diagnosed with terminally ill colon cancer and needs resuscitation. He did not sign a DNR order. The physician has decided to perform a 'slow code' on his own. Is this the right decision? (Important)
Answer: no. Please remember, a performance of 'slow code' or 'show code' is not acceptable to the patient. This decision by the doctor represents the failure to come to a timely and clear decision about the patient's resuscitation status.
70. A 20-year old man is diagnosed with suspected bacteremia and meningitis. He refuses antibiotic therapy. He collapses and requires resuscitation. What should a physician do in this situation?
Answer: the physician should resuscitate the patient despite the patient's refusal to antibiotic therapy.
71. A 50-year old woman is diagnosed with severe aortic stenosis. She collapsed in a doctor's office and is required resuscitation. She is waiting for valve replacement surgery. What should a physician so in this situation?
Answer: this condition is called 'physiological futility'. In severe aortic stenosis, vigorous resuscitation is highly unlikely to restore adequate cardiac output. Therefore, the physician might reasonably refrain from resuscitation.
72. A 14-year old boy is diagnosed with terminally ill cancer. He is not responding to chemotherapy. His parents want to continue the treatment. However, the boy does not want to continue his suffering. The physician told the parents that chemotherapy will not be helpful. What is the appropriate decision at this point? (Important)
Answer: the boy's decision should be respected because the treatment is futile.
73. A surrogate pregnant mother made a surrogacy contract with a couple in which she will give the baby to the couple. She developed complications in the first trimester and wanted to abort. Is she allowed to do that? (Important)
Answer: yes. If her life or health becomes threatened from continuing the pregnancy, she should retain her right to abortion.
74. A physician became sexually involved with a current patient who initiated or consented to the contact. Is it ethical for a physician to become sexually involved?
Answer: no. Sexual involvement between physicians and former patients raises concern. The physician should discuss with a college or other professional before becoming sexually involved with a former patient. The physician should terminate the physician-patient relationship before initiating a romantic or sexual relationship with a patient.
75. A physician decided to take care of his own family members and relatives. He is a very smart physician. Is this a right decision?
Answer: no. The physician should encourage all friends and family members to have their own personal physician.
76. A male patient wants to have a copy of his medical records. What should a physician do in this situation?
Answer: the physician should retain the original of the chart. Information should only be released with the written permission of the patient or the patient's legally authorized representative ( e.g., attorney).
77. A 30-year old female wants to have an abortion. Her physician objects to abortion on moral, religious, or ethical grounds. What should a physician do in this situation? (Important)
Answer: physician should not offer advice to the patient.
78. A physician sees patients at a reduced fee. He spends very little time with each patient. Is the physician doing the right thing?
Answer: no. The physician is not providing optimal care.
79. A surrogate pregnant mother signed a surrogacy contract with a couple. Male partner gave sperms which were artificially inseminated to the surrogate mother. Surrogate mother has a genetic relation to the child. She wants to void the contract after the baby is born. Is she allowed to breach the contract? (Important)
Answer: yes. Surrogate contracts, while permissible, should grant the birth mother the right to void the contract within a reasonable period of time after the birth of the child.
80. A surrogate pregnant mother signed a surrogacy contract with a couple. Both male and female parents gave sperm and ovums respectively. The surrogate mother wants to void the contract and she has no genetic relation. Is she allowed to breach the contract? (Important)
Answer: no. Genetic parents have exclusive custody and parental rights.
81. A surrogate pregnant mother signed a surrogacy contract with a couple. The couple got divorced. Male partner gave sperms and the female partner gave ovums. They do not want to continue the surrogate pregnancy. What should be the decision at this point? (Important)
Answer: the couple is genetically related to the fetus. They have the right not to continue with this pregnancy.
82. A surrogate pregnant mother signed a surrogacy contract with a couple. The couple got divorced. Male partner gave sperms but the female partner could not give ovums. They do not want to continue the surrogate pregnancy. What should be the decision at this point? (Important)
Answer: female partner has no right to terminate this pregnancy because she has no genetic relation. Surrogate mother has genetic relation and she has the right to continue this pregnancy even if the male partner disagrees.
83. A 3-year old girl is diagnosed with blood cancer. She has been waiting for an umbilical cord transfusion. Her mother delivered a newborn baby girl. Umbilical cord blood was obtained and was transfused to the 3-year old sibling. What is the duty of the physician?
Answer: physician should obtain an informed consent of the risks of donation and he or she should follow the normal umbilical cord clamping protocol. Physician should protect both the children.
84. A 31-year old man has decided to donate one of his kidneys for a large amount of money. Is this the right decision?
Answer: no. However, the donor can receive some payment to cover his medical expenses. Only the potential donor not the donor's family or another third party may accept financial incentive. Payment should occur only after the organs have been retrieved and judge medically suitable for transplantation.
85. A couple has decided to have a child through artificial insemination. They asked the physician for sex selection of the child. What should a physician advise in this situation? (Important)
Answer: physician should not participate for sex selection for reasons of gender preference. However, sex selection of sperm for the purpose of avoiding a sex-linked inheritable disease is appropriate.
86. A 30-year old man has donated his sperms which were kept frozen. He died in a car accident. He did not leave any specific instructions regarding sperm donations. His wife wants to make use of them. A woman requested her to donate his sperms. What is the appropriate decision? (Important)
Answer: the donor's wife can use the semen for artificial insemination but not to donate it to someone else. The donor should give clear instructions at the time of donation. The donor has the power to override any decision.
87. The donor and recipient of sperms are not married. Who would be considered the sole parent of the child? (Important)
Answer: the recipient. Except in cases where both donor and recipient agree to recognize a paternity right.
88. The residents and medical students were asked by an attending to follow certain orders for a patient. The residents and medical students believe the orders reflect serious errors in clinical or ethical judgment. What is the appropriate way to handle the situation? (Important)
Answer: The residents and medical students should not follow those orders. They should discuss with the attending issuing those orders. They should also discuss the situation with a senior attending physician, a chief of staff, or a chief resident.
89. A physician used a newly prescribed drug to his patient. The patient got sick after the drug was ingested and required hospitalization. Should the physician report this drug's side effect to FDA (Food and Drug Administration)? (Important)
Answer: yes. FDA should be notified only if the drug causes serious adverse events such as those resulting in death, hospitalization, or medical or surgical intervention.
90. A 39-year old female has been suffering from chronic cholecystitis. The surgeon advised cholecystectomy. The patient wants a second opinion. The surgeon agreed. The patient went to another surgeon and has decided to be operated by the second surgeon. What should the second surgeon do in this situation? (Important)
Answer: the second surgeon should accept the patient because the patient has the right to choose the surgeon. First surgeon should accept the patient's decision.
91. A 45-year old male was admitted to the hospital with mild chest pain. He wants to leave the hospital before completion of therapy. How do you manage the patient? (Important)
Answer: The patient is asked to sign a statement that he is leaving against medical advice (AMA). The patient may however leave without signing that statement. This document is a legal evidence that the patient was warned by the physician about the risk of leaving. Please remember, discharge AMA does not apply to children.
92. A 55-year old man requested his physician to misrepresent his medical condition to receive disability or insurance payment. What is the appropriate response of the physician in this situation?
Answer: The physician must refuse that request.
93. What is the responsibility of a fellow physician who is aware of drug abuse, alcohol abuse, or psychiatric illness of his colleagues or of a medical condition that is harmful to patients? (Very Important)
Answer: the physician should protect the patients. The fellow physician should report to the appropriate authority ( i.e., report to the hospital authority; report to the Dean for a medical student's problem).
94. A 60-year old male has been suffering from severe pain due to terminal prostate cancer. The patient is receiving lower doses of narcotics and sedatives. How can a physician relieve his suffering? (Important)
Answer: The physician should increase the dosage of narcotics and sedatives up to the maximum recommended amount. The suffering can be reduced by listening, spending more time with him, and reducing psychological distress.
95. A 25-year old female medical student or resident noticed a mistake made by a junior attending physician during rounds. She is afraid of that attending physician. What is the appropriate way to handle the situation? (Important)
Answer: She should discuss the situation with a more senior attending physician for appropriate interpretation, advice, and assistance.
96. A 26-year old male medical student or resident made a mistake during patient care. He is afraid of what might result. What is the appropriate way to handle the situation?
Answer: He should disclose the mistake to the attending physician and try to learn from that mistake. The patient should be notified as well.
97. A 63-year old female health care worker is concerned about taking care of patients with HIV infection or multidrug-resistant tuberculosis. What is the appropriate way to handle the situation? (Important)
Answer: The physician should provide appropriate care to patients despite personal risk. Institutions should reduce the risk of infection by appropriate equipment, supervision, and training. Her concern should be taken seriously.
98. A 30-year old male physician has an opportunity for financial incentive if he sees more patients and refers them unnecessarily. What is you opinion about this?
Answer: The physician should provide only care that is in the patient's best interest.
99. Two physicians are discussing a case inside the elevator of a hospital. What is your opinion about this?
Answer: They should not do that because they have to maintain the patient's confidentiality.
100. The patient's confidentiality should be maintained except in the following situations:
Physicians should override third parties in case of domestic violence, child abuse, elderly abuse, gunshot wounds, syphilis, and tuberculosis. They should report these cases to appropriate governmental authorities.
101. A physician is experiencing a very difficult ethical issue regarding a complicated case. He is confused. What should be the next step?
Answer: he should discuss the matter with other faculty members in his health care team, colleagues, or hospital ethics committee.
102. What is the final plan of action in an ethical issue?
Answer: Both patient and physician should agree regarding final management. The patient should be well-informed about the medical condition. The physician should be sympathetic and knowledgeable regarding the relevant medical condition.
103. DNR (do not resuscitate) order. This is appropriate if the patient or surrogate signed that order or if CPR (cardiopulmonary resuscitation) would be futile. Physicians should write DNR orders and the reason for them in the chart. Please remember, "slow" or "show" codes are not acceptable. Foods and fluids are considered therapies that should be stopped. (Important)
104. (A)Brain death (adult): (Important)
(i) Definition by the President's Committee:
Death is an irreversible cessation of circulation and respiratory functions or irreversible cessation of all functions of the entire brain and brain stem.
(ii) The criteria of brain death by the staff of Massachusetts General Hospital and the Harvard Committee:
Death occurs when there is absence of all signs of receptivity, responsivity, and all brainstem reflexes, and the EEG is isoelectric. Sometimes metabolic disorders and intoxications may mimic the above findings.
(iii) The guidelines of brain death:
(a) The diagnosis should be made also by another physician and confirmed by clinical findings and EEG.
(b) The family should be notified. They should not make the decision about discontinuing medical treatment except in a situation where the patient has directed the family to make the decision.
© The physician should discuss with another physician before removing supportive measures (e.g., ventilators).
(d) Family members may request organ donation, and in many states physicians may request the family to make an organ donation.
(B) Brain death (children):
(i) Definition: same as in adults.
(ii) Criteria: similar in children and adults, but the period of observation is longer in children.
Children 1wk-2mo of age: two separate examinations 48 hours apart
Children 2mo-1yr of age: two separate examinations 24 hours apart
Children more than 1 yr of age: two separate examinations 12 hours apart
Spontaneous movements must be absent, with the exception of spinal cord reflex withdrawal and myocolnus.
Generalized flaccidity should be present. The presence of clinical criteria for 2 days in term and 3 days in preterm infants indicates brain death in majority of asphyxiated newborns. The absence of cerebral blood flow on radionuclide scan and silence of electrical activity on EEG are not always observed in brain-dead newborns. There is no universal consensus about the definition of neonatal brain death. The decision is made after discussion with the family and health care team. If there is difference of opinion, the ethics committee should be consulted. The decision is made on the basis of what is in the best interest of the infants and children.
105. Practice guidelines for physicians:
(i) The best way to practice medicine is to select useful diagnostic techniques and therapeutic measures which are most appropriate to a particular patient and clinical condition.
(ii) Practice guidelines can reduce the health care costs, which improves health care to patients who even do not have adequate health care benefits.
(iii) Please remember, guidelines do not and should not be the only way of managing an individual patient.
106. Some important points about patients:
(i) For a patient with an incurable disease, the major goal of therapy should be the enhancement of the quality of life.
(ii) The patient care begins with a personal relationship between the patient and the physician. If a patient has confidence on the physician, reassurance may be the best therapy. The patient must understand that the physician is giving the best possible care available.
107. Patients who do not have decision-making capacity about their medical care:
The patients who do not have decision-making capacity usually arrange for surrogates who make decisions for them. Their choices depend on their values. Psychiatrists are helpful in mentally impaired patients. Family members are usually the surrogates, because they know the patients very well
Advanced directives: statements made in advance in case patients lose their decision-making capacity in the future. These directives indicate the names of surrogates and which interventions are acceptable or not acceptable to them. These are achieved by oral conversation (most common form), living will, health care power of attorney, or physicians can ask the patient in advance.
Absence of advance directives and surrogates: physicians can make the decision using all information and should respect the patient's values. Physicians must know the laws of the state in which they practice.
Patient preferences are known:
The decision is made with the patient's best interest in mind.
Disagreements between potential surrogates or between the physician and surrogate: Physicians can consult with the hospital ethics committee or with other physicians. The courts should be only the last resort.
108. Down syndrome with different medical conditions: (Very Important)
(a) Duodenal atresia at birth: surgical repair is recommended as it is done regularly.
(b) VSD (ventricular septal defect) in newborn period: initial conservative medicals management is followed by surgical repair as it is done regularly.
© Cyanotic heart disease at birth: immediate medical management, which is followed by surgical repair as it is required routinely.
(d) Cosmetic surgical condition (e.g., rhinophyma or big nose): there is no urgency to repair the underlying condition, but it can be done as it is performed regularly.
(e) Neural tube defects (e.g., meningomyelocele): surgical repair is recommended as it is done regularly.
Please remember, a patient's management should be discussed with his/her parents and the decision made with the best interest of the patient in mind.
109. A healthy male patient with Down Syndrome lives independently. He went to a doctor for facial cosmetic surgery. Can he make his own decision? (Important)
Answer: yes. The patient can make his decision if he understands the procedure and the consequences. He lives independently which indicates that he is capable of making his own decision.
110. A patient went to the doctor due to throat pain. The doctor asked the patient what her problem was. The patient said that she woke up at six o'clock in the morning, went to the bathroom, ate breakfast, and went to drop her children at school. She then came back home, stared cooking and continues to talk about irrelevant things. What should the doctor do to stop the patient from rambling? (Important)
Answer: the doctor should ask the patient to tell him what problems she has related only to her throat.
111. A patient went to a doctor for abdominal pain but remained quiet throughout the visit. He did not tell the doctor enough about his symptoms. What should the doctor do? (Important)
Answer: the doctor must ask the patient detailed questions about his abdominal pain. It is the doctor's obligation to find out as much as he can about the patient. Without enough information, the doctor will not be able to make an accurate diagnosis.
112. A patient walked into his doctor's office with acute abdominal pain. He has been suffering from ulcerative colitis. The patient is noncompliant and did not visit for the past six months. What should the doctor do in this situation? (Very Important)
Answer: The doctor should find out more about the patient's abdominal pain before making any other decision. The doctor should always be responsible with the patients.
113. A terminally ill pancreatic cancer patient with multiple metastasis is admitted to the hospital. He is in critical condition. The patient wants to know his prognosis. What should the doctor say?
Answer: the doctor should tell the patient politely that he will discuss his condition with him and his family. The doctor should never specify the longevity of the patient. The doctor should tell the truth even when the patient is a child. The doctor should not hide any medical information from the patient.
114. A patient is recently diagnosed with cancer. He is nervous but is eager to know about his medical diagnosis. What should the doctor's reply be? (Important)
Answer: the doctor should gently tell the patient his condition.
115. A patient is recently diagnosed with cancer. Previously, he had an episode of a nervous breakdown after hearing a family death and had to be admitted to a hospital. He loves his family members and tends to be very open with them on all issues. How should the doctor tell the patient about his current state? (Important)
Answer: the doctor should call his family members and discuss the patient's medical condition openly and politely.
116. A male patient was admitted with severe myocardial infarction. He was admitted to the ICU and his condition is very critical. He does not know the reason for his admission. The patient is unstable. What should the doctor tell the patient? (Important)
Answer: the doctor should wait until the patient is stabilized and then gently tell him his medical condition.
117. A mother gave birth to a premature baby who was admitted to the NICU (neonatal intensive care unit). The baby is on a mechanical ventilator. The mother wants to hold the baby. What should the doctor do in this situation? (Important)
Answer: the mother should be allowed to hold the baby.
118. A male patient is recently diagnosed with HIV. Should the doctor ask about his sexual orientation (i.e., male, female, or both)? (Important)
Answer: yes, the doctor should ask the patient directly but politely about his sexual orientation.
119. A homosexual male patient went to a doctor. The patient's partner was recently diagnosed with HIV. Should the doctor ask the patient whether he is being penetrated by his partner or he penetrates his partner?
Answer: yes, because the person who is being penetrated has a higher incidence of HIV due to trauma in perianal area.
120. A 6-year old boy comes to the ER after drowning. He expired in the ER despite appropriate resuscitations. The family members became angry which is a reflection of a sense if guilt and helplessness. What is the appropriate way of giving information to the family members?
Answer: the physician should give the information clearly and compassionately when there is no hope for survival. Parents need to know that everything was done to save the child.
121. A pregnant woman who is Rh(-)ve became sensitized. She had H/O induced abortions. Her husband is not aware of his wife's previous abortions. He wants to know from the physician how she became sensitized. (Very Important)
Answer: the physician should tell the man to ask his wife. The physician should not mention anything about the patient's H/O abortions.
122. A mother brought her infant to the ER. The radiologist test reveals old fractures of the ribs. She did not know anything about that. (Very Important)
Answer: this is a case of child abuse. This case should be reported to child welfare agency.
123. A physician is examining a child with respiratory distress. The child's mother became anxious during the physical examination. Please remember, a patient's management should be discussed with his/her parents and the decision made with the best interest of the patient in mind. (Important)
Answer: child abuse.
124. A mentally retarded patient became pregnant. The patient does not want an abortion. Her mother and husband want an abortion. What should a physician do in this situation? (Important)
Answer: abortion should not be performed.
125. A male physician is examining an adolescent or adult female patient. What should a physician do in this situation? (Important)
Answer: a chaperone should be present during the physical examination. The same rules apply when a physician is examining a patient who appears to be seductive. (Important)
126. A female physician is examining an adolescent or adult male patient. What should a physician do in this situation? (Important)
Answer: a chaperone should not be present during the physical exam.
127. A suspected HIV patient expired in a car accident. He signed for organ donations. What should a physician do in this situation?
Answer: his HIV status is not certain. The organs can be preserved until the HIV status is confirmed. If the test for HIV is positive, organs should be discarded.
128. A patient who expired in a car accident signed in his license foe organ donations. His license has expired. He always wanted to donate his organs. What should a physician do in this situation? (Important)
Answer: physican cannot accept organs because the signed consent has expired.
129. A male physician sexually harassed a female patient during the physical examination. The patient complained to a nurse. What should the nurse do in this situation?
Answer: the nurse should tell the patient to make an official report to the hospital authority or to an appropriate agency.
130. A chronic male smoker comes to the physician for his heart problems. The physician wanted his patient to quit smoking. What should the physician advise in this situation?
Answer: the physician should ask the patient to quit smoking immediately because patients are usually more responsive when they are ill. The physician should assist the smoker to move one step closer to quitting.
131. A terminally ill patient did not sign a DNR (do not resuscitate) order, however, he signed a DNI (do not intubate) order. What should the physician do in this situation?
Answer: the physician should follow his orders i.e., the patient should be resuscitated but should not be intubated, despite severe hypoxic condition of the patient.
132. A terminally ill patient signed a DNR order, however, he did not sign a DNI (do not intubate) order. He wants to be intubated but not resuscitated. What should a physician do in this situation?
Answer: the physician should follow his orders i.e., the patient should be intubated but should not be resuscitated.
133. An adolescent car accident victim was brought to the ER in an unconscious state. The patient needs immediate surgical interventions. The surgeon was unable to contact any family member to obtain consent. What should a surgeon do in this situation? (Important)
Answer: the surgeon should do the procedure without waiting to obtain a consent for the benefit of the patient.
134. An obgyn doctor is recently diagnosed with HIV infection. He is receiving medication for HIV. His physical and mental conditions are normal. Should he tell his patients or fellow physicians about his HIV status?
Answer: no, however, the doctor should take appropriate precautions for infection control. He does not have to tell his fellow physicians about his HIV status including the physicians who are referring patients to him. The doctor is allowed to see patients if he takes appropriate precautions. However, he should notify the hospital authority.
135. A physician is scared of seeing an HIV patient with an open wound. Can a physician refuse to see a patient?
Answer: yes, however, a physician's refusal to see a patient is unethical but is legal.
136. An elderly semicomatose patient may require surgical intervention. His family members are confused about the surgery. They asked the surgeon for his opinion. What should the surgeon's response be?
Answer: the surgeon can give his opinion and act as a moral surrogate for the benefit of the patient. (Very Important)
137. A 12-year old boy is diagnosed with a terminal illness (e.g., malignancy). He asked the doctor about his prognosis. His parents requested the doctor not to tell him the bad news. What should the doctor do in this situation? (Very Important)
Answer: the doctor should tell the truth politely and compassionately to the patient.
138. A 55-year old woman is recently diagnosed with right breast cancer. The doctor told the patient that she will require surgery for removal of the right breast. She started to cry. What should a doctor do in this situation?
Answer: first, the doctor should give her some tissue paper for wiping her tears. Then, the doctor should be sympathetic to her and console her. He might tell her that similar reactions are usually expected from other patients with breast cancer. Please do not mention that she will be fine with a breast implant or without a right breast because she is already 55-years old.
139. A mother is carrying a 500 gram premature fetus which develops acute fetal distress. The physician wanted to perform a cesarean section. Mother refused cesarean section. What should the doctor do in this situation?
Answer: the physician should arrange a bedside conference with the mother along with other physicians, social worker, and administrator to discuss the matter.

ETHICS PROBLEMS
Reply
#36
RADIOLOGY
http://www.med-ed.virginia.edu/courses/r...index.html
http://www.neuroanatomy.wisc.edu/natbrdrev/nbrbase1.htm
Reply
#37
UW notes..these are not mine, but ere very useful!!

Pediatrics:

1. With supracondylar fractures, the brachial artery can be compromised, resulting in the loss of the radial artery pulse; therefore, the radial artery pulse must be assessed when the fracture is reduced. Supracondylar fractures are the MC fractures in the pediatric population.

2. Pure riboflavin (vit. B2) deficiency is unusually in industrialiaed nations, but has been documented in regions of the world with severe food shortages. The condition is typically mild and nonspecific in presentation, but symptoms may include sore throat, hyperemic and edematous oropharyngeal mucous membranes, cheilitis, stomatitis, glossitic, normocytic-normochromic anemia, seborrheic dermatitis, and photophobia.

3. Ascorbic acid deficiency (Vit C): ecchymoses, petechiae, bleeding gums, hyperkeratosis, Sjogren™s syndrome, arthralgias, impaired wound healing. Systemic manifestations include: weakness, malaise, joint swelling, arthralgias, edema, coiled hair, depression, neuropathy, vasomotor instability.

4. Pellagra: (niacin deficiency): symmetric reddish rash present in exposed areas of skin, a red tongue, nonspecific symptoms such as diarrhea, vomiting, insomnia, anxiety, disorientation, delusions, dementia, encephalopathy. May be observed in alcoholics, long-trem users of isoniazid, and those stricken with carcinoid syndrome or Hartnup disease.

5. Leiomyomas: MCC of hysterectomy, Malignant transformation is extremely rare.

6. Depression: is a valid reason for a patient to be considered incompetent to make decisions. Refer the patient for psychiatric evaluation of depression first, then treat the patient.

7. Retinal detachment: usually painless, sudden onset, seeing flashes of light, seeing floaters of black spots in the field of vision, unilateral involvement.

8. Asthma:
 Acute attack: β2-antagonist
ï‚· Chronic obstructive pulmonary disease exacerbation: Ipratropium
ï‚· Prophylaxis: cromolyn and zafirlukast
ï‚· Prednisone doesn™t provide immediate relief but reduce inflammation several hours later and often are useful adjuncts to β2-antagonist

9. Thiamine deficiency: associated with beriberi, Wernicke-Korsakoff syndrome. Manifestations of infantile beriberi appear between the ages of 2 and 3 months, and include a fulminant cardiac syndrome with cardiomegaly, tachycardia, cyanosis, dyspnea, and vomiting.

10. Adult beriberi: is categrorized as dry or wet.
ï‚· Dry beriberi is characterized by a symmetrical peripheral neuropathy accompanied by sensory and motor impairments, especially of the distal extremities.
ï‚· Wet beriberi includes neuropathy in addition to cardiac involvement (e.g. cardiomegaly, cardiomyopathy, congestive heart failure, peripheral edema, tachycardia)

11. Stranger anxiety: is the normal anxiety experienced by infants when they are exposed to unfamiliar individuals. It peaks at 12-15 months.

12. Acute post-streptococcal GN: is the MC form of GN in children. It may occur following pharyngitis or pyoderma.
ï‚· HTN+edema
 ↓ C3 and CH50, but C4 is normal (this indicates the activation of the alternate complement pathway).-hypocomplementemia resolves in 8-12 weeks.
ï‚· hematuria, proteinuria (renal function returns to normal in 1-2 weeks)
 ↑ ASO titer.

13. Bed rest with the hip joint in a position of comfort is the Tx of choice for transient synovitis of the hip. (the condition will get better in 3-4 days)

14. Meingococcemia: suspect ~ in a neonate with signs of meningitis and a petechial rash. H.Influenza may cause meningitis but without the rash. Listeria can cause meningitis but without the rash either. GBS can casue meningitis and with a rash, but it is very rare in 18 M infant, but shortly after birth.


15. Myotonic muscular dystrophy (MMD): an AD disease which is known as Steinert disease. The 2nd MC muscular dystrophy in the U.S. The pathology is distinct in that all types of muscles (ie. Smooth, striated, cardiac) are involved. Presentations: muscle weakness, progressive muscle wasting, posterior forearm muscles, anterior compartment of the lower legs. Temporal wasting, thin cheeks, an upper lip in the shape of an inverted V. Myotonia is defined as delayed muscle relaxation, and the classic example is the inability to release the hand after a handshake.

16. Causes of amblyopia: --std Tx: occlusion of the normal eye.
ï‚· Strabismus (MCC)
ï‚· Errors of refraction
ï‚· Opacity of media along the visual axis

17. Clubfoot is initially managed with stretching and manipulation of the foot, followed by serial plaster casts, malleable splints, or taping. Surgical Tx is indicated if conservative management gives unsatisfactory results, and is preferably performed between 3 and 6 months of age. (mild atrophy of the calf, calcaneum and talus are in equines and varus positions, midfoot is in varus position, forefoot in adduction. Dorsiflexion and plantar flextion of the ankles are limited. Normal neurologic examination.)

18. Todd™s paralysis: represents a postictal condition that usually rapidly improves with restoration of motor function within 24 hours. (hemiparesis. Sudden loss of consciousness with following disorientation and slow gain of consciousness is a characteristic description of a seizure, if the convulsive episode was missed.) It may follows a generalized as well as focal seizures.

19. Neuroblastoma: the 3rd MC cancer in the pediatric population (after leukemia and CNS tumors). Arise from neural crest cells, which are also the precursor cells of the sympathetic chains and adrenal medulla. Calcifications and hemorrhages are seen on plain X-ray and CT scan. The levels of HVA and VMA are usually elevated. May arise from adrenal or any location along the paravertebral sympathetic chains. Easy to metastasize.

20. Psoriasis: more common in white, positive family history, pitting of the fingernails, arthritis (DIPs always involved).

21. Whenever IV access can™t be obtained in emergent pediatric cases, intraosseous access should be attempt next.


22. Guillain-Barre syndrome: suspect ~ in a child who presents with an ascending polyneuropathy one week after apparent viral infection. The underlying pathology involves mainly the peripheral motor nerves, although sensory and automomic nerves may also be affected. Associated with Campylobacter jejuni enteritis. Tx. plasmapheresis, immunoglobin (IV), no prednisone here (ineffective and may prolong recovery.

23. Child developments:
Language development:
ï‚· Social smiles: 2 M
ï‚· Babbles: 6 M
ï‚· 2-words, obeys 1-step command: 1 Y
ï‚· 2-3 phrases, obeys 2-step command: 2 Y

Gross motor development:
ï‚· Holds head: 3M
ï‚· Rolls back to front and front to back: 4 M
ï‚· Sits well unsupported: 6 M
ï‚· Walks alone: 1 Y
ï‚· Walks up and down stairs without help: 2 Y

Fine motor development:
ï‚· Raking grasp: 6 M
ï‚· Throw object: 1 Y
ï‚· Build tower of 2 blocks: 15 M
ï‚· Build tower fo 6 blocks/turn pages of books: 2 Y

Social development:
ï‚· Recognizes parents: 2 M
ï‚· Recognizes strangers (stranger anxiety): 6 M
ï‚· Imitates action/comes when called: 1 Y
ï‚· Plays with other children : 18 M
ï‚· Parallel play: 2 Y

24. Osgood-Shlatter disease: typical patient is a 10-17 yo boy with knee pain at the tibial tuberosity, which is the site of insertion of the quadriceps tendon.

25. Septic joint in a child is a true surgical emergency and needs immediate surgical drainage. A delay of even 4-6 hours can lead to avascular necrosis of the femoral head.

26. Subarachnoid hemorrhage (SAH): can be cuased by an intraventricular hemorrhage, which is common in premature infants. Accumulation of the blood in the subarachnoid space may lead to destruction of the arachnoid villi and cisterns, thereby blocking the flow or decreasing the absorption of CSF and leading to communicationg hydrocephalus. SAH is the MCC of communicating hydrocephalus.

27. MCC of syncope: vasovagal.

28. Sudden infant death syndrome (SIDS): is the leading cause of mortality in infants between 1 M and 1 Y, and the 3rd MCC of mortality in infants <1 Y.

29. Perinatal problems: is the leading cause of mortality in infants < 1 M.

30. Drugs/diseases associated with hirsutism:
ï‚· Minoxidil (anti HTN agent, used to treat alopecia)
ï‚· Polycystic ovary syndrome
ï‚· Cushing syndrome
ï‚· cyclosporine

31. Pyloric stenosis: a 4-6 week old infant with projectile vomiting that worsen over time. Peristaltic waves are seen over the upper abdomen, and an olive-sized mass is palpated. Tx: surgical correction.

32. Vesicourteral reflux: Reflux is a risk factor for UTI. Repeated attacks can lead to progressive renal scarring, which is the major cause of end stage renal disease and HTN in children. Dx of VUR is best made with a voiding cystourethrogram (VCUG) or a radionucleide cystogram (RNC). Suggested after 1st UTI.

33. Bedwetting: is considered normal until the age of 4-5 yrs. If nocturnal enuresis persists, DDAVP (DOC) or imipramine may be used.

34. Mild protein intolerance: should be suspected when a neonate presents with bloody diarrhea (maroon-colored), eosinophils in the stool, and a positive family history of atopy. (e.g. mom is asthmatic)

35. Precocious pubarche with signs of severe androgen excess is suggestive of precocious pseudo-puberty that is casued by a gonadotropin-independent process (typically an excess of sex steroid, such as in a late onset congenital adrenal hyperplasia-21 hydroxylase deficiency)

36. Tuberous sclerosis (infantile spasm): best Tx is im ACTH. Presents with: episodes of jerkey movements of the neck, arms, and legs onto the trunk, occur in clusters, last for a few minutes, often preceded by a cry. Multiple small 1-2 cm oval irregular hypopigmented macules, ash-leaf spots, shagreen spots (orange peel lesions, sebaceous adenomas on her trunk and extremities. A head CT: cortical tubers in the cerebral cortex and multiple subependymal nodules in the lateral ventricles. EEG: hypsarrhythmia. Chromosome 9 & 16

37. An immediate anaphylactic reaction, an encephalopathy, or any CNS complication within 7 days of administration of the vaccine is a contraindication for further administration of DTaP. In these instances, DT should be substituted for DTaP since the adverse reactions are usually attributed to the pertussis component of the vaccine. *extremely HY Q for USMLE*

38. Congenital anomalies are the 2nd leading cause of death in infants < 1 Y.

39. Cyclical vomiting: recurrent self-limiting episodes of vomiting and nausea in children, in the absence of any apparent cause, suggest the Dx. Its incidence is high in children whose parents have a history of migraine.

40. Chlamydia: is the MC causative agent of infectious neonatal conjunctivitis. Chlamydial pneumonia can develop in infected infants (congenital chlamydial infection).

41. Friedreich Ataxia (FA): autosomal recessive disease. (triplet repeats-tocopherol transfer protein abnormality): progressive with poor prognosis. ↓ vibratory and position sense in the lower extremities, feet are deformed with high plantar arches. MRI of the brain and spinal cord shows marked atrophy of the cervical spinal cord and minimal cerebellar atrophy. Nerve conduction velocity normal. EKG shows T-wave inversions in the inferior and lateral chest leads.

42. T wave inversion differentials:
ï‚· MI
ï‚· Myocarditis
ï‚· Old pericarditis
ï‚· Myocardial contusion
ï‚· Digoxin toxicity

43. Tx of choice for local impetigo: topical mupirocin, or, oral erythromycin.

44. Febrile seizure: Consider the Dx of ~ when an otherwise healthy child presents with a fever and isolated seizure. These seizures are benign and mangaged with antipyretic therapy.

45. Pubertal gynecomastia: is seen in approximately one-half of adolescent boys, at an average age of 14 years. It is often asymmetric or transiently unilateral, and frequently tender. In prebubertal males the testicular size is normally 2 cm in length and 3 mL in volume. The initial management involves reassurance and watchful waiting/observation. *extremely HY Q for USMLE*

46. APGAR:
Color of the newborn:
0 body and extremities are blue/pale
1 body is pink and extremities are blue
2 body and extremities are pink

Heart rate:
0 heart shows no activity
1 HR <100
1 HR > 100

Reaction to nasal stimulation:
0 no response
1 grimace
2 active cough

Tone/Activity
0 limp
1 some flextion of extremities
2 active flextion of extremities

Respirations:
0 completely absent
1 slow and irregular
2 good respiratory effort

47. Craniopharyngioma: a young boy with symptoms of increased intracranial pressure (headache, vomiting), bitemporal hemianopsia, and a calcified lesion above the sella has a ~ until proven otherwise. Presence of a cystic calcified parasellar lesion on MRI is diagnostic of craniopharyngioma.

48. Tetralogy of Fallot: the MC cyanotic congenital disease in children less than 4 yo, presents with cyanotic spells and pansystolic murmur on examination.
ï‚· Overringding aorta
ï‚· Right ventricular hypertrophy
ï‚· Subpulmonary stenosis (single S2)
ï‚· VSD (pansystolic murmur)

49. Turner™s syndrome with 46 XY karyotype: is associated with a higher incidence of gonadoblastoma; hence, prophylactic bilateral gonadectomy is indicated in the management of such patients.
50. Endocrinology:

1. A 35yo white male presents with fatigue, decreased appetite, weight gain, constipation and cold intolerance. He can™t recall any stressful event. He does not take any medications. No smoking no alcoholic. HR 47, BP:145/91mmHg.PE revels cool, pale skin, coarse hair, and brittle nails. There is delayed relaxation of DTR. The thyroid gland is normal on palpation. Lab studies reveal increased serum free T3 and T4 level, and normal serum TSH level. Which of the following is the most likely Dx.?
A. Primary hypothyroidism
B. Secondary hypothyroidism
C. Subclinical hypothyroidism
D. Generalized resistance to thyroid hormones.
E. Graves™ disease

Answer is D. Patients with generalized resistance to thyroid H. have high serum T4 and T3 levels with normal to mildly elevated TSH levels. Patients typically have features of hypothyroidism despite having elevated free thyroid hormones.

2. Elevated serum DHEA-S (Dehydroepiandrosterone-sulfate) levels are specifically seen in patients with androgen-producing adrenal tumors.

3. A 40 yo asymptomatic male comes to the office for a routine physical examination. His serum chemistry panel shows:
Sodium:140 mEq/L, Potassium 4.0 mEq/L, Bicarbonate 25 mEq/L, chloride 101 mEq/L Calcium 11.6 mg/dL Phosphorus 2.2 mg/dL
24-hr urine collection reveals a calcium level of 200 mg, and creatinine level of 1.5 g. Bone mineral density by dual energy X-ray absorptiometry (DXA) shows normal bone mineral density. Neck examination reveals no masses. What is the most appropriate next step in the management of this patient?
A. Bisphosphonate therapy
B. Surgical exploration of the neck
C. Medical surveillance
D. Loop diuretics
E. Thiazide diuretics
Explanation: B. The patient most likely has asymptomatic primary hyperparathyroidism. Laboratory findings of hypercalcemia, hypophosphatemia and elevated PTH levels are very suggestive of the Dx. Asymptomatic primary hyperPTH is a common disorder, particularly in females over 60 yo. Most patients are usually identified during routine chemistry screening. While surgical intervention is needed for all patients with symptomatic primary hyperPTH, not all asymptomatic patients require such as. The indication for surgery in asymptomatic patients is the presence of at least one of the following features:
1. Serum Cacium level at least 1mg/dL above the upper normal limit with urinary calcium excretion greating than 50mg/24hr
2. Urinary calcium excretion greater than 400 mg/24hr.
3. Young patients (<50 yro)
4. Bone mineral density lower than T-2.5 at any site.
5. Difficulty in follow-up of the patient.

4. Spot urine collection and times urine collection for the measurement of urine microalbumin to creatinine ratio are generally accepted as good screening methods for microalbuminuria. Although 24-hr urine collection is slightly more accurate in screening for microalbuminuria, its inconvenience to patients makes it less preferred by physicians.

5. Suspect DKA in stuporous patients with rapid breathing and a history of weight loss, polydipsia and plyuria.

6. Hyperpigmentation of the skin and mucous membranes is characteristic of primary adrenocortical deficiency, and is due to the increased levels of ACTH (MSH like). This clinical feature is not seen in patients with secondary adrenal insufficiency, which is due to hypothalamo-pituitary failure.

7. The most likely Dx. In a patient who presents with clinical features of adrenal insufficiency and calcifications in the adrenal glands is adrenal tuberculosis. Adrenal tuberculosis continues to be the prominent cause of primary adreanal insufficiency in developing countries. In contrast, autoimmune adrenalitis is currently the MCC of primary adrenal insufficiency in developed countries.

8. Suspect the following conditions whenever a patient presents with hypokalimia, alkalosis and normotension.
Surreptitous vomiting
Diuretic abuse
Bartter syndrome
Gitelman™s syndrome
Physical findings that are characteristic of surreptitious vomiting are scars/calluses on the dorsum of the hands, and dental erosions, hypovolemia and hypochloremia, which in turn lead to a low urine cl concentration.

Always suspect surreptitious vomiting as a cause of hypokalemic alkalosis in a normotensive patients, and be able to distinguish it from other entities (e.g. diuretic abuse, Bartter™s syn.) using the urine chloride concentration.

9. .Diabetes insipidus presents as poly uria, polydipsia, and excretion of dilute urine in the presence of elevated serum osmolarity.
. Primary polydipsia is due to excessive water drinking; both plasma and urine are diluted.
. SIADH results in hyponatremia, low serum osmolality and inappropriately high urine osmolality.

10. α-glucosidase inhibitors block dietary carbohydrate breakdown in the intestinal tract. The most significant side effects are GI disturbances due to the increased undigested carbohydrate concentration in the stool.

11. Metabolic acidosis observed during diabetic ketoacidosis is typically accompanied by hyperkalemia; this is sometimes called paradoxical hyperkalemia because the body potassium reserves are actually depleted. (extremely HY Q for the USMLE!)

12. The serum albumin level should always be measured simultaneously with the serum calcium level in order to calculate the correct total serum calcium value. With every 1 g/dL change in serum albumin, serum calcium changes by 0.8 mg/dL.

13. Nonketotic hyperosmolar syndrome (NKHS) occurs in type 2 DM because the level of insulin in these patients is sufficient to prevent ketosis, but not hyperglycemia. In most cases, severe hyperglycemia develops, thereby resulting in glycosuria and severe dehydration. The severe hyperosmolality is also responsible for lethargy, weakness, altered mental status, focal neurological deficits and eventual coma.

14. Fasting blood glucose measurement is now the recommended screening test for DM. 126 mg/dL on two separate occasions is diagnostic of DM.

15. Untreated hyperthyroid patients are at risk for rapid bone loss resulting from increased osteoblastic activity in the bone cells. Untreated hyperthyroid patients are also at risk for cardiac tachyarrhythmias, including atrial fibrillation.

16. Increased extracellular pH levels (e.g. respiratory alkalosis) can cause an increase in the affinity of serum albumin to calcium, thereby increasing the levels of albumin-bound calcium, and consequently decreasing the level of ionized calcium. Ionized calcium is the only physiologically active form, which means that decreased levels of this form can result in clinical manifestations of hypocalcemia.

17. Important causes of thyrotoxicosis with low radioactive iodine uptake include:
ï‚· subacute painless thyroiditis
ï‚· subacute granulomatous thyroiditis
ï‚· iodine-induced thyroid toxicosis
ï‚· levothyroxine overdose
ï‚· struma ovarii

18. Arterial pH or anion gap is the most reliable indicator of metabolic recovery in patients with diabetic ketoacidosis.

19. Suspect primary hyperaldosteronism in a young patient with hypertension, muscle weakness and numbness; the most specific lab value for the patient with primary hyperaldosteronism is high aldosterone/rennin ratio, indicating automomic aldosterone secretion.

20. MEN II consists of medullary carcinoma of the thyroid (MTC), hyperparathyroidism and pheochromocytoma. The serum calcitonin level is elevated in patients with medullary thyroid cancer. Virtually 100% of patients with MENII have c-cell hyperplasia or MTC, 50% have pheo, and 20-30% have hyperparathyroidism.

21. In the treatment of a patient using both sildenafil and an alpha-blocker, it is important to give the drugs with at least a 4-hour interval to reduce the risk of hypotension.

22. In patients with MEN IIa syndrome, genetic testing has replaced biochemical measurement of serum calcitonin as the recommended screening test. If genetic analysis is positive for a RET proto-oncogene mutation, total thyroidectomy is indicated.




GIT
a. Laxative abuse is characterized by very frequent (10-20), watery, nocturnal diarrhea. The Dx. can be confirmed with the characteristic biopsy finding of dark brown discoloration of the colon with lymph follicles shining through as pale patches (melanosis coli)

b. CT scan is the best test for the Dx. of diverticulitis in acute setting.

c. Suspect ischemic colitis in patients who have evidence of atherosclerotic vascular disease, present with abdominal pain followed by bloody diarrhea, and have minimal abdominal exam findings. The most commonly involved segment of the colon is the splenic flexure, because it is supplied by end arteries.

d. Always suspect Crohn™s disease in a young patient with chronic bloody diarrhea.

e. Know the stepwise approach of the Tx for ascites:

1. Sodium and water restriction
2. Spironolactone
3. Loop diuretic (not more than 1 L/day of diuresis)
4. Frequent abdominal paracentesis (2-4 L/day, as long as the renal function is okay)

b. Most colon cancers develop from polyps. The risk factors for a polyp progressing into malignancy are villous adenoma, sessile adenoma, and size >2.5 cm. Only adenomatous polyps are clearly premalignant, but <1% of such lesions progress to malignancy. Hyperplastic polyps are non-neoplastic and do not require further work-up. (extremely HY for USMLE)

c. Suspect celiac disease in any patient who presents with malabsorption and iron deficiency anemia. Celiac disease (sprue) is associated with anti-endomysial antibodies.

d. D-xylose absorption is abnormal both in bacterial overgrowth and Whipple™s disease. However, with bacterial overgrowth, the test becomes normal after antibiotic Tx.

e. Bacterial overgrowth is a malabsorption syndrome which can be associated with a history of abdominal surgery.

f. The initial Tx of both acute and chronic anal fissures includes dietary modification (e.g. high-fiber diet and large amounts of fluids), a stool softener, and a local anesthetic.

g. Suspect tropical sprue in patients with malabsorption, along with a history of living in endemic areas for more than one month. Tropical sprue involves the small intestine; the typical biopsy is characterized by blunting of villi with infiltration of chronic inflammatory cells, including lymphocytes, plasma cells and eosinophils.

h. The two MCC of painless GI bleeding in an elderly patient over 65 years of age are diverticulosis and angiodysplasia (vascular ectasia). There is a well defined association between aortic stenosis and angiodysplasia.

i. Iron deficiency anemia is one of the MC presentation of celiac sprue. Upt to 24% of cases of celiac disease is associated with dermatitis herpetiformis.

j. Drug-induced pancreatitis is mild and usually resolves with supportive care. CT scan is diagnostic for pancreatitis. Remember the following scenarios for drug-induced pancreatitis:
ï‚· Patient on diuretics? Furosemide, thiazides
ï‚· Pateint with inflammatory bowel disease? Sulphasalazine, 5-ASA
ï‚· Patient on immunosuppressive agents? Azathioprine, L-asparaginase
ï‚· Patient with a history of seizures or bipolar disorder? Valproic acid
ï‚· AIDS patient? Didanosine, pentamidine
ï‚· Patient on antibiotics? Metronidazole, tetracycline

k. The BUN level is often elevated in patients with upper GI bleeds because the bacterial breakdown of Hb in the GI tract results in the absorption of urea. A BUN level > 40 in the presence of a normal serum creatinine level may include an upper GI bleed. The other common scenario where you can see elevated BUN without increased creatinine levels is the administration of steroids.

l. Currently, quantitative estimation of stool fat is the gold std for the Dx of steatorrhea. (72 hrs fecal fat collection)

m. Esophagoscopy is indicated when a patient with gastroesophageal reflux disease (GERD) fails to respond to empiric Tx., or when a patient has features of complicated disease.

n. Abdominal CT scan is the next Dx test when abdominal ultrasound doesn™t explain cholestatic jaundice.

o. No matter what the underlying disease of the patient is, remember that the management of any patient arriving in the ED begins with assessment of the patients™ œABCs.

p. Noninvasive tests for H. pylori should be the first step in the management of patients with dyspepsia under age 45 who do not have alarming symptoms.

q. Manometry (esophageal motility studies) establishes the Dx of diffuse esophageal spasm.

r. Carcinoids are most commonly found on the appendix; however, patients who present with carcinoid syndrome usually have carcinoids located in the small bowel.

s. Recognize when to order upper endoscopy in GERD:
ï‚· Nausea/vomiting
ï‚· Weight loss, anemia or melena/blood in the stool
ï‚· Long duration of symptoms (>1-2 yrs), especially in Caucasian males >45 yo
ï‚· Failure to respond to proton pump inhibitors

t. Remember the pancreatic cholera-VIPoma

u. A combination of hepB virus immune globulin and lamivudine is the most effective measure to prevent recurrent HBV infection after liver transplantation. But it is not effective in fulminant hepatitis.

v. Whipple™s disease can mimic many illnesses, such as hyperthyroidism, connective tissue disease, alcoholism and AIDS. It should be suspected in all patients with fever of unknown origin (FUO), generalized lymphadenopathy, arthralgias, weight loss, abdominal pain and diarrhea. Dx of Whipple™s disease can be readily made using upper gastrointestinal endoscopy and PAS-staining of the obtained small intestinal biopsies. The classical findings are PAS-+ material in the lamina propria and villous atrophy.

w. Painless jaundice in an elderly patient should make you think about pancreatic head carcinoma.

x. Increased intragastric pressure during vomiting can cause tears in the mucosa of the cardia, and sometimes of the distal esophagus. These are called Mallory-Weiss tears

y. Lactose intolerance is characterized by a positive hydrogen breath test, a positive Clinitest of stool for reducing substances, and an increase stool osmotic gap.

z. The extent of a malignancy determines the most appropriate, timely, and individualized patient care. CT is a std Dx tool employed in patients with newly diagnosed gastric ca. to evaluate the extent of the disease. Surgical removal of the affected tissueds remains as the mainstay of therapy.

aa. MI is one of the differential Dx of acute abdominal pain and should be ruled out in patients with risk factors. (ECG)

Infectious Diseases

1. Mitral regurgitation is the most common valvular abnormality observed in patients with infective endocarditis not related to IV drug abuse.

2. Any HIV-positive patient with bloody diarrhea and normal stool examination should have a colonoscopy and biopsy done to look for CMV colitis. CMV colitis is characterized by bloody diarrhea with abdominal pain, multiple ulcers and mucosal erosions on colonoscopy; biopsy shows characteristic cytomegalic cells with inclusion bodies.

3. Diarrhea, in HIV-infected patients, can be due to multiple etiological agents therefore; and etiologic Dx must be made before starting antibiotic therapy. (stool examination for Salmonella, Shigella, Campylobacter, Clostridium difficile, Giardia, Cryptosporidium, Mycobacterium avium complex and CMV.

4. A 32 yo male comes to his physician with a 2week history of fatigue, fever, muscle and joint pains. His vitals are, BP:115/75 mmHg;T: 37.7 C; RR: 14/min; PR: 75/min. Physical examination is unremarkable, except for splenomegaly. Lab studies show: Hb: 13 gm/dL; WBC count: 15,000/microL; Neutrophils: 42%; Lymphocytes: 50%, monocytes: 5%, Basophils: 1%; Eosinphils: 2%; Platelet count: 300,000/microL. Large basophilic lumphocytes with vacuolated appearance are seen. Monospot test is negative. What is the most likely cause of this patient™s symptoms?
a. chronic fatigue syndrome
b. CMV infection
c. Acute toxoplasmosis
d. Acute retroviral syndrome
e. Chronic lymphocytic leukemia
Explanation: The patient described in this vignette has a mononucleosis-like syndrome in which atypical lymphocytes are found in the blood. Atypical lymphocytes are large basophilic cells with a vacuolated appearance. They may be found in CMV infection, acute toxoplasmosis, and acute retroviral syndrome, but CMV infection is the MC of all the listed causes.

For a Dx of chronic fatigue syndrome to be made symptoms must be present for over 6 months.

In CML, lymphocytes are small, mature, and they constitute 70-80% of WBCs.

Educational objective:
Blood smear with atypical lymphocytes should make you rank CMV higher on the list in a patient with mononucleiosis-like symptoms.

5. Condyloma Lata and bilaterally symmetrical maculopapular rash involving the entire trunk and extremities are characteristic of second stage of syphilis. Remember the rash is present on palms and soles. Serological tests are positive in secondary syphilis.

6. IV cefotetan, ampicillin/sulbactam, or the combination of clindamycin and a fluoroquinolone is the appropriate empirical Tx for limb-threatening infections in diabetics, whereas mild, or non-lim-threatening, infections can be treated with oral antibiotics like cephalosporin, clindamycin, amoxicillin/clavulanate and fluroquinolones.

7. Always consider malaria in patients from endemic areas with high-grade periodic fever and chills. Anemia and splenomegally are the clinical clues.

8. Proteus is the most likely cause of urinary tract infection in patients with alkaline urine.

9. In a HIV patient, bilateral interstitial pneumonia is most likely due to Pneumocystis carinii infection.

10. VDRL testing, PPD skin testing, Hep A and B serology and ab titer for Toxoplasma are indicated as a part of initial work-up in all newly diagnosed HIV-positive patients.

11. A nail puncture wound in an adult resulting in osteomyelitis is most likely due to Pseudomonas aeruginosa. (Clostridium tetani doesn™t cause ostomyelitis)

12. Mucormycosis requires aggressive surgical debridement plus early systemic chemotherapy with amphotericin B.

13. Whenever a health care worker is exposed to HIV, baseline HIV testing should be performed immediately and postexposure prophylaxis with combination of two or three antiretroviral drugs should be started without any delay.

14. In case of suspected oestomyelitis, blood cultures and x-rays should be taken and the patient should be started on IV antibiotics. If the x-ray are negative, three-phase technetium bone scan should be considered. Needle biopsy can be done to identify the organism if the blood cultures are negative.

15. Streptococcus bovis endocarditis is associated with colorectal cancer and colonoscopy is advisable in such patients.

16. Untreated LGV (lymphogranuloma venereum) caused by Chlamydia trachomatis serotypes L1-3) may progress to a severe and chornic disease causing ulceration (painless), proctocolitis (inflammation of colon and rectum), rectal stricture, rectovaginal fistulas and elephantiasis.

17. Any patient from southwestern region with history of tick bite developing systemic symptoms along with leucopenia and thrombocytopenia should make you think about Ehrlichiosis. (Tx. Doxycycline)

18. Enterotoxigenic E. coli is the most frequent offender causing travelers diarrhea. Travelers with abdominal cramps, diarrhea and malaise should be suspected for infection by this organism.

19. Always consider the risk of splenic rupture in case of infectious monomucleosis (IM) and advise the patient to avoid contact sports to prevent this hazard. (no exercises till PE is normal)-atypical lymphocytes, EBV. Heterophil antibody test is sensitive and specific for Dx. of IM

20. Cutaneous larva migrans (creeping eruption, is a helminthic disease casued by infective stage larva of dog or cat hookworm, also called Ancylostoma braziliense.) is a common cause of dermatological disease in travelers from tropical regions, and is characterized by pruritic elevated serpiginous lesion on the skin.*Remember sand box handling*

21. Urethral cultures have higher yield than synovial or blood culture in cases of suspected gonococcal purulent arthritis.

22. Consider CMV pneumonitis as a late complication in post BMT recipients with dyspnea and cough (CT: ground glass attenuation and innumerable small nodules.)

23. Osteomyelitis in DM that involves the bone adjacent to the foot ulcers is explained by the contiguous spread of infection.

24. Intermittent catheterization is an effective measure to reduce the risk of UTI in patients with neurogenic bladder.

25. Patients with hemochromatosis are vulnerable to listeria monocytogenes infections and some other bacterial infections. Know the various bugs that are likely to jeopardize such patients.

26. Campylobacter is the MCC of bloody diarrhea in USA. Presence of severe abdominal pain along with diarrhea is a helpful clue.

27. Albendazole or mebendazole is the first line Tx. For E. vermicularis infection (pinworm). Pyrantel palmate is an alternative.

28. Vibro paraheamolyticus is usually transmitted by ingestion of seafood. Patients having symptoms of food poisoning after intake of seafood should be suspected for this.

29. Steptococcus pneumoniae is the MC pathogen causing pneumonia in nursing home patients.

30. Patients with trichinosis, presents with GI complaints followed by muscle pain, swelling, and weakness. Presence of subungual splinter hemorrhages, conjunctival and retinal hemorrhages, periobital edema and chemosis should make you think about trichinosis.

31. Pseudomonas is commonly responsible for nosocomial pneumonia in intubated patients. Cefepime or ceftazidime are the commonly used medications.

32. HIV infected patients who develop esophagitis are first started on fluconazole directed against candidiasis.

33. Glucocorticoids are indicated in a case of IM complicated by upper airway obstruction, autoimmne hemolytic anemia, and thrombocytopenia.

34. Gonorrhea is a common organism,which causes STD. It is also a common cause of pharyngitis, generally acquired from oral sex.

35. Immunocomplex disease is primarily responsible for glomerulonephritis, Roth spots and Osler™s nodes. Janeway lesions result from septic embolism. * Extrememly HY Q for USMLE.

36. The Tx. Of PCP is always triemthoprim-sulfamethoxazole. However, the second choice agent is pentamidine.

37. Always suspect endocarditis whenever a patient is febrile and has other constitutional features in the presence of a new heart murmur

38. Anti TB therapy should always be supplemented with vitamin (pyridoxine) to avoid neurological complications.






Poisoning

1. Duration of QRS complex is the best measure for assessment of severity of tricyclic antidepressant toxicity. (Toxicity with TCA is characterized by anticholinergic effects and QRS widening on EKG.)

2. Pyridoxine (Vit. B6) is an antidote for isoniazid.

3. Bromocriptine is used in the Tx. of neuroleptic malignant syndrome, which is characterized by marked hyperthermia, muscular rigidity, tremors, altered mental status, and diaphoresis.

4. Phenothiazine cause hypothermia by causing vasodilatation and by inhibition of shivering.

5. Methanol intoxication is associated with visual loss. Ethylene glycol poisoning is associated with renal failure and crystalluria. Methanol is not hepatotoxic.

6. Suspect caustic poisoning in a conscious patient with a white tongue, heavy salivation, mouth burns, drooling of saliva and dysphagia. The patient is usually in severe pain.

7. Impaired concentration and conjunctival injection are important features of marijuana use. Also, it causes behavioral changes and 2 or more of the followings: dry mouth, tachycardia, increased appetite.

8. Spider bites:
ï‚· Acute abdomen is a feature of black widow bite and is best treated with a combination of calcium gluconate and musle relaxants.

ï‚· Brown recluse spider bites produce an extensive localized skin necrosis resembling a pyoderma gangrenosum. Dapsone is used to reduce the extent of local necrosis in patients who have been screened for glucose-6-phosphate dehydrogenase (G6PD) deficiency.

9. Sodium bicarbonate prevents the development of arrhythmia in patients with TCA by alleviating cardio-depressant action on sodium channels.

10. Calcium is useful in reversing cardia effect of calcium channel blocker.

11. Flumazenil, a benzodiazepine antagonist, is antidote for acute benzodiazepine intoxication.

12. Alcohol competes with CYP2E1, so in acetaminophen intoxication, results in decrease production of toxic metabolite.

13. Decision of use of N acetyl cysteine as an antidote for acetaminophen overdose is generally based on 4 hour post-ingestion acetaminophen levels. Gut emptying procedures are best effective if carried out in the first hour.

14. Dextromethorphan interacts with monoamine oxidase inhibitors and can produce severe hyperthermia.

15. Diphenhydramine toxicity produces seizures as well as anti-cholinergic effects.

16. Chlordiazepoxide is the treatment of choice for delirium tremens that is characterized by disorientation, hallucination, tachycardia, hypertension, and agitation (such as alcohol withdrawal).

17. Magnesium is an effective Tx for torsade de pointes.

18. First step in the management of pesticide poisoning is to remove the source of poison.

19. Sodium bicarbonate is effective for the Tx of cardiac dysfunction induced by thioridazine toxicity.

20. Pinpoint pupils and respiratory depression are the hallmark features of acute opioid toxicity for which naloxone is the drug of choice.

21. Contrast studies with gastrograffin are indicated in addition to upper gastrointestinal endoscopy when a patient with acute alkali ingestion is suspected of having esophageal perforation.


Preventive Medicine

1. Bupropion has been approved by FDA for smoking cessation program.

2. Dysthymia= depressed mood for most days for at least two years.

3. Pap smear: if 3 consecutive pap smears are normal, screening may be performed less frequently (every 3 years) in a low-risk patient. Screening is usually started at 18 and stopped at 60-75 years.

4. Patients with egg allergy can have severe allergic reaction or anaphylaxis with influenza, yellow fever or MMR vaccine.

5. Hepatocellular cancer is a vaccine-preventable cancer (hepatitis B vaccine)

6. Hepatitis A is the MC vaccine-preventable disease among travelers. It should be considered in people who are planning to visit developing countries.

7. The USPSTF recommends that screening for lipid disorders should include measurement of total cholesterol (TC) and HDL with fasting or non-fasting samples.

8. USPSTF recommends the use of total cholesterol and HDL cholesterol for the purpose of screening, however, for treatment purposes, the recommendations are based on total risk assessment and LDL cholesterol levels.

9. Female patients over the age of 50 are required to undergo annual mammograms until the age of 75. In general, screening studies are not routinely recommended for people older than 80.

10. All adults should be immunized against diphtheria and tetanus every 10 years.

11. USPSTF strongly recommendes routine screening of male at/above 35, and females at/above 45 for lipid disorders. * HY Q for USMLE.

12. Human studies have demonstrated a significant association between type A personality and exaggerated cardiovascular response.

13. Women who (1)are immunocompromised, (2) had in utero exposure to DES, or (3) have a history of CIN II/III or cancer should have annual Pap smear, even if their prior Pap smears are negative.* HY Q for USMLE.

14. Influenza vaccine is recommended on an annual basis for all adults above 65 yo, and adults of any age who are at risk for developing complications from influenza infection (as listed above).

15. Even though MMR is a live attenuated vaccine, it should be given to all HIV patients who are not severely immunocompromised.


Cardiology

1. The hyperdynamic type of septic shock is characterized by
an elevated cardiac output
low systemic vascular resistance, right artrial pressure and pulmonary capillary wedge pressure, (PCWP)
a frequently normal mixed venous oxygen concentration.

2. Diagnose right ventricular infarct, which should always be suspected in the setting of an inferior wall MI, with hypotension. Understand its pathophysiology and hemodynamics.

3. A clear association has been found between excessive alcohol intake and development of HTN. ( greater than smoking effect).

4. Clopidogrel should be included as secondary prevention following UA/NSTEMI for at least 12 months. It should also be prescribed for 30 days (bare metal stent) to one year (drug eluting stents) following PCI, as it has been shown to help prevent subacute stent thrombosis.

5. Clopidogrel + apspirin: is more effective than aspirin alone for the first 30 days following percutaneous coronary interventions (PCI), as it helps prevent subacute stent thrombosis. Patients who receive drug-eluting stents need a longer duration because epithelialization occurs slowly.

6. Know how to Dx restrictive cardiomyopathy in the setting of amyloidosis. Speckled pattern on echocardiogram is very specific for amyloidosis.

7. Restrictive cardiomyopathy: characterized by severe diastolic dysfunction due to a stiff ventricular wall. Chest X-ray shows only mild enlargement of the cardiac silhouette. Echo usually shows a symmetrically thickened ventricle wall, normal or slightly reduced LV size and normal or near normal systolic function. Kussmaul™s sign may also be present. The apical impulse is easily palpable in restrictive cardiomyopathy as opposed to constrictive pericarditis.

8. Restrictive cardiomyopathy is difficult to differentiate from constrictive pericarditis. With constrictive pericarditis, chest X-rays may show pericardial calcifications and the CT scan usually shows increased thickness of pericardium. Kussmal™s sign may be positive in both contitions. With constrictive pericarditis, the thickness of myocardium will be normal.

9. Mitral valve prolapse (MVP) is the MCC of mitral regurgitation in U.S.A.

10. MVP symptoms: substernal chest pain not related to exertion or ingestion of food. Palpitation, Murmur type: mid systolic click with late systolic crescendo-decrescendo murmur. Click and murmur occurs earlier with Valsava maneuver and it disappears with squatting. EKG is normal. Best Tx for this chest pain: β-blockers.

11. β-blockers are used to treat chest pain, palpitation, and autonomic symptoms of MVP. ˜

12. Murmur of MVP: mid systolic click with late systolic crescendo-decrescendo murmur. Click and murmur occurs earlier with Valsalva maneuver and it disappears with squatting. EKG can be normal.

13. The mechanism of mitral regurgitation in HOCM is the systolic anterior motion of mitral valve leaflet.

14. First-degree heart block is a completely benign arrhythmia and requires no Tx. (eg. HR 68, PR interval >0.2)

15. Know how to manage a case of CHF exacerbation due to A. Fib with a rapid ventricular response. Digoxin is the DOC in this situation. Also anticoagulant should be given, as this is one of the most important interventions in reducing the morbidity and mortality associated with atrial fibrillation.

16. Atrial fibrillation in MS is due to left atrial dilatation. (history of Rhematic fever, mid diastolic rumble, and loud S1) Left atrial enlargement that results from MS predisposes the patient to the development of AFib.

17. The hallmark of MS: elevated left atrioventricular pressure gradient.

18. Antihypertensive management should be the first step in patients with aortic dissection with HTN. Transesophageal echocardiogram (TEE) is the initial investigation of choice to Dx suspected aortic dissection. But it is indicated after HTN Tx is initiated.

19. Recognize infective Endocarditis in a patient who has recently undergone GU instrumentation (cytoscopy).

20. It™s better to keep systolic pressure<130 mmHg to slow end-organ damage in patients with diabetes and chronic renal failure. 120/80 mmHg is considered optimal in DM patients.

21. Even though IV β-blockers improve mortality in acute MI, they are contraindicated in the presence of pulmonary edema. (severe shortness of breath, bilateral crackles half way up to the lungs. Obvious jugular venous distension and 2+ pedal edema bilaterally). This case, DOC is furosemide.

22. Thiazide diuretics have some unfavorable metabolic side effects including hyperglycemia, increased LDL cholesterol, and plasma triglycerides. Electrolyte abnormalities that can be induced by thiazide diuretics include hyponatremia, hypokalemia, and hypercalcemia (good for kidney stone patient, though).

23. The diagnosis of ventricular septal rupture can be made if there is evidence of left to right shunting on Swan-Ganz catheter readings, when a 2D-echo is not available.

24. Recognize the clinical presentation of pulmonary edema. Iatrogenic fluid overload is one of the common causes of pulmonary edema in perioperative patients. (increased urine volume is an evidence)

25. Emphasize the importance of CK-MB for the Dx of recurrent myocardial infarction. It begins to rise within 4-5 hours after MI and returns to baseline within 48-72 hours. It™s high specificity and rapid return to the baseline makes it the biomarker of choice for the Dx. of a recurrent MI. (CK-MB fraction has a high specificity for an acute MI (slightly lower than cardiac troponins).

26. Primary biochemical tests used for the Dx of acute MI: Cardiac troponins T and troponins I. They begin to rise 4-6 hours after an MI, and remain elevated for 10 days. They have also replaced LDH for the retrospective Dx of MI. But because of their persistent elevation for 10 days after an MI, they can™t be used to establish the Dx of re-infarction within 1-2 weeks after an MI.

27. Tobacco and alcohol are reversible risk factors for the development of atrial premature beats.

28. GI endoscopy is a low-risk procedure for infective endocarditis. For GI endoscopy, prophylaxis is optional in high-risk patients and not recommended in moderate-risk patients.
High risk to develop infective endocarditis:
ï‚· All prosthetic heart valves.
ï‚· Any history of previous bacterial endocarditis
ï‚· Complex cyanotic congenital heart disease and surgical constructed systemic pulmonary shunts.
Moderate risk to develop infective endocarditis:
ï‚· Congenital cardiac malformations not falling into the high or negligible risk categories (such as PDA, VSD, Ostium primum ASD, bicuspid aortic valve and coarctation)
ï‚· Acquired valvular heart disease (such as rheumatic heart disease, valvular stenosis and reguregitation)
ï‚· MVP with regurgitation and/or myxomatous leaflets.
ï‚· Hypertrophic cardiomyopathy.

Conditions that do not require IE prophylaxis include:
ï‚· Isolated ostium secundum ASD and surgically-reparied ASD, VSD and PDA (beyond six months and without sequelae)
ï‚· MVP without mirtral regurgitation and without thickened leaflets.
ï‚· Innocent or physiologic murmurs (echo required in the adult population to rule out valvular lesion).
ï‚· Cardiac pacemakers and defibrillators.
ï‚· History of isolated bypass surgery, history of Kawasaki disease without valvular dysfunction and history of rheumatic fever without valvular dysfunction.

29. Syncopal episode without following disorientation (post-episode confusion is more characteristic for a seizure), hearing impairment, normal PE, and family history of sudden cardiac death should make you think of congenital long QT syndrome. Beta-blockers are the DOC.

30. Aspirin, ACEIs and beta-blockers have been shown to reduce mortality in the setting of acute MI.

31. Know how to recognize and treat right ventricular infarction. IV NS to increase the outflow from right ventricle should be considered in these patients (avoid lowering preload, stop nitrate, give a normal saline bolus).

32. Recognize the early complications of an acute anterior wall MI: mitral regurgitation-- papillary muscle dysfunction, or rupture, is the MCC of MR in this setting.

33. When A. fib is associated with hemodynamic compromise, cardioversion is the treatment of choice.

34. Thrombolytic therapy is not indicated for unstable angina or non-ST elevation (non Q wave) MI. The treatment of unstable angina primarily includes aspirin, beta-blockers, heparin and nitroglycerin.

35. In all cases of ST elevation MI, reperfusion therapy with thrombolytics or PTCA (PCI) with or without stenting must be performed ASAP. PTCA (PCI) is preferred over thrombolytics.

36. Reentrant ventricular arrhythmia (ventricular fibrillation) is the MCC of death in patients with acute myocardial infarction.

37. The Tx of ventricular fibrillation is STAT defibrillation with 200-360 joules. If defibrillation fails, lidocaine or aminodarone (DOC) can be loaded and the patient shocked again. Epinephrine can sensitize the heart and lower the threshold for conversion.

38. Think of PE in a postoperative patient with JVD and new onset RBBB.

39. Amiodarone has the potential to cause lung fibrosis and should be avoided in patients with history of pulmonary fibrosis.

40. Beta-blockers have been shown to decrease the risk of perioperative coronary events.

41. It is important to recognize that oral contraceptives can be a potential cause of HTN, and simply discontinuing its use can correct the problem. It causes hypercoagulable stage too.

42. The MCC of aortic dissection is systemic HTN.

43. Calcium channel blockers do not improve survival in patients with acute MI (such as nifedipine). On the other hand, aspirin, thrombolytics, ACEIs and β-blockers have been shown to improve survival in patients with acute MI.

44. First degree heart block is characterized by a prolonged constant PR interval (>0.2s). There will not be any dropped beat.


45. Wenckebach or Mobitz type I heart block is characterized by a narrow QRS, progressive increase in PR interval until a ventricular beat is dropped, then the sequence is repeated. It is a benign arrhythmia and is transient. Unless the patient is symptomatic, it requires no Tx.

46. Mobitz type II heart block is a dangerous arrhythmia which can progress to complete heart block and requires a permanent pacemaker.

47. Morbitz type II block, the PR interval remained unchanged prior to the œP wave and it suddenly failed to conduct to ventricles. So you will see a dropped œQRS complex with normal œPR interval.

48. In third degree AV block, no atrial impulses will travel to ventricles. So atria and ventricles beat independently and have their respective rates.

49. In AFib, the heart rate is irregular and you will not see any P waves.

50. Suspect aortic dissection as a cause of tearing chest pain in the setting of HTN and BP difference in the 2 arms.

51. The measurement of serum BNP can help distinguish between CHF and other causes of dyspnea. A value>100 pg/mL Dx CHF with a sensitivity, specificity, and predictive accuracy of 90,76, and 83 percnet, respectively.

52. Choose the appropriate initial antihypertensive therapy in a patient with asthma. Hydrochlorothiazide is the initial DOC in patients with chronic persistent asthma.

53. Recognize the high risk of arterial thromboembolism associated with anterior wall MIs (LAD, left anterial descending branch). Inferior wall MI is associated with a right ventricular infarction in more than 1/3 of cases. Lateral wall MIs, posterior wall MIs, and right ventricular infarctions are not associated with an increased risk of arterial thromboembolism.

54. TB is the MCC of constrictive pericarditis, in immigrant population. It should be considered in patients with unexplained elevation of JVP and history of predisposing condition.

55. High-dose niacin therapy that is used to treat lipid abnormalities frequently produces cutaneous flushing and pruritis. This side effect is explained by prostaglandin-induced peripheral vasodilation and can be reduced by low-dose aspirin.

56. Descending aortic aneurysm in a young male is usually due to blunt trauma to the chest.

57. Decrease in the synthesis of non-cholesterol products may be responsible for some adverse effects of statin therapy; for example, reduced CoQ10 production is implicated in the pathogenesis of statin-induced myopathy.

58. In the treatment of cocaine-related cardiac ischemia, the first-line drugs are benzodiazepines, nitrates, and aspirin. (IV diazepam)

59. Systolic heart failure is characterized by depressed cardiac index (CI) accompanied by increased total peripheral resistance (TPR) and left ventricular end-diastolic volume (LVEDV).

60. Stress EKG or an exercise echocardiogram should be considered for risk stratification in patients with stable angina.

61. It is important to look for and treat hemochromatosis early in patients who present with restrictive cardiomyopathy, because this intervention significantly improves prognosis in these patients. (avoid restrictive cardiomyopathy if early Tx is started).

62. Always suspect malignant HTN in patients with very high BP (>=200140 mmHg). Presence of papilledema on ophthalmoscopy confirms the Dx. The pathologic change responsible for end-organ damage in malignant HTN is fibrinoid necrosis of small arterioles.

63. Think of cocaine intoxication in a young patient presenting with chest pain/myocardial infarction or stroke. Features of cocaine intoxication are cocaine bugs, agitation, decreased appetite, dilated pupils, elevated or decreased BP, tachycardia or bradycardia, and sweating.

64. Patients with artificial pacemakers and defibrillators do not require prophylaxis for infective endocarditis.

65. Suspect aortic dissection in a patient with acute retrosternal pain and a normal EKG. Check BP in both arms and auscultate for DM of aortic regurgitation. Transesophageal echocardiography is the preferred diagnostic tool. Before performing the TEE, HTN should be controlled.

66. Lidocaine is not used prophylactically in patients with acute coronary syndromes. Although its use decreases the risk of ventricular fibrillation, it may increase the risk of asystole.

67. Electrical alternans is an important EKG finding for the Dx of pericardial tamponade. Other findings include sinus tachycardia and low voltage QRS complexes.

68. EKG:
ï‚· T wave inversions occur with ischemia of the myocardium.
ï‚· ST segment depression occurs with subendocardial infarcts and unstable angina.
ï‚· Prolonged PR interval occurs in cases of first-degree heart block and it is not an EKG finding of cardiac tamponade.
ï‚· Delta waves are present in Wolff-Parkinson-White syndrome.
ï‚· New onset of RBBB is seen in right ventricular strain, especially with massive pulmonary thromboembolism.

69. Dihydropyridine CCA can cause peripheral edema and should always be considered in the DD of this condition, along with other causes, such as heart failure, renal disease and venous insufficiency.


70. The investigation of choice for the Dx of HCM is echocardiography.

71. Murmur type of HCM( hypertrophic cardiomyopathy)-a harsh crescendo-decrescendo grade III SM heard most prominently along the left lower sternal border. The murmur is intensified ↑ by Valsalva maneuver and attenuated ↓ by leg elevation)-less blood, more murmur; more blood less murmur----characteristic of HCM.

72. Bicuspid aortic valves represent the MCC of aortic stenosis in middle age adults.

73. Senile calcific aortic stenosis is the MCC of aortic stenosis in elderly (60-80 yo).

74. Presence of hypokalemia and HTN warrants investigations for secondary cause. Measure plasma renin activity and serum aldosterone level.

75. Elderly patients are particularly sensitive to fluid loss, and even mild hypovolemia may predispose them to orthostatic syncope, especially upon getting up in the morning. BUN/Creatinine ratio is a useful indicator of dehydration.

76. Left ventricular aneurysm can cause CHF in a patient who sustained an anterior wall MI in the past. A double apical beat and persistent elevation of the ST segment are important diagnostic clues.

77.

78. Bluish discoloration and cool fingers in the ICU are a common finding after use of norepinephrine (pressor Tx.) for hypotension.

79. Manage a patient with ST segment elevation MI with immediate angiography and PTCA when thrombolytic are contraindicated. Even if the patient has no contraindications for thrombolytic therapy and a catheterization lab is available in the hospital, or within 30 mins of the hospital, PTCA with stent placement has been shown to have better outcomes than thrombolytic therapy in acute ST elevation MI.

80. Prophylaxis drugs for IE:
ï‚· Amoxicillin is the DOC for prophylaxis of IE in dental and respiratory procedure. In patients who are allergic to penicillin, alternatives include cefazolin, clarithromycin or clindamycin.
ï‚· For genitourinary and GI procedures, other than esophageal procedures, the regimen of choice is ampicillin plus gentamicin in high-risk patients. If the patient is allergic to penicillin, a combination of vancomycin + gentamycin is used in high-risk patients. (if allergic to penicillin, give no ampicillin or amoxicillin?)

81. Hypertrophic cardiomyopathy is the MCC of sudden cardiac death in young athletes.

82. Isolated systolic HTN is an important cause of HTN in elderly patients. It is created by decreased elastic properties of the arterial wall. Always treat isolated systolic HTN, in spite of the fact that diastolic BP is not elevated. The DOC would be hydrochlorothiazide.

83. Isolated diastolic dysfunction: secondary to hypertrophic cardiomyopathy. The cause of hypertrophy cardiomyopathy (essential HTN caused). Features suggesting isolated diastolic dysfunction of the LV are the normal cardiac size, the normal ejection fraction and normal LV EDV, as well as the presence of an S4 gallop. DOC: β-blockers as they improve diastolic filling by lowering the HR and increasing the diastolic filling time. They also reduce the myocardial oxygen demand and cause regression of LV hypertrophy due to reduction of BP.

84. Vasovagal syncope is the MCC of syncope. It is frequently recurrent. Upright tilt table testing may be indicated to confirm the Dx if the syncope is recurrent.

85. The most likely culprit lesion for acute inferior wall myocardial infarction is right coronary artery (RCA) occlusion, especially if it is complicated by right ventricular infarction (hypotension) and bradycardia.

86. Cacium channel blockers (diltiazem) are the DOC for variant angina.

87. Nitrates are contraindicated when a patient is continuously or intermittently taking sildenafil (Viagra). Should not be given to the patient within 24 hours of the last dose of sildenafil. Otherwise, nitrate will cause syncope, MI or sudden death. They both induce nitric oxide mediated vasodilatation.

88. Clopedogrel should be included as secondary prevention following UA/NSTEMI for at least 12 months. It should be prescribed for 30 days (bare metal stent) to one year (drug eluting stents) following PCI, as it has been shown to help prevent subacute stent thrombosis.

89. Inferior wall myocardial infarction: can result from the occlusion of either the right coronary artery or the left circumflex artery. (RCA: bradycardia and hypotension, involvement of SA node and right ventricle.)

90. The earliest EKG finding in acute MI is peaked (hyperacute) T waves, followed by ST segment elevation, followed by the inversion of T waves, followed by the appearance of Q waves. The earliest changes of hyperacute T waves are frequently not seen in clinical practice because by the time the patient present they already have ST elevation.

91. Diffuse ST segment elevation is seen in: pericarditis, pulmonary edema

92. Acute pericarditis: typical findings are
ï‚· chest pain, worsened by breathing and improved by leaning forward,
ï‚· presence of a pericardial friction rub
ï‚· Diffuse ST segment elevation that is concave upwards.

93. Depressed CO combined with elevated PCWP (an indicator of left atrial pressure, and most of the times left ventricular end diastolic pressure) is indicative of left ventricular failure.--à cardiogenic shock.

94. Age-dependant idiopathic sclerocalcific changes are the MCC of isolated aortic stenosis in elderly patients. These changes are common and usually have minimal hemodynamic significance, but sometimes may be severe.

95. Aortic stenosis: systolic ejection murmur and soft S2, displaced apical impulse is due to the hypertrophied LV. Most appropriate investigation would be and echocardiogram to confirm the Dx.

96. Torsades de pointes is an arrhythmia of gradually changing QRS morphology and most often caused by Quinidine. In the acute setting magnesium replacement is the cornerstone of the Tx.

97. Heparin should be started in all unstable angina patients. Immediate angiography and possible revascularization are indicated when ischemia is refractory to optimal medical Tx or when there is evidence of hemodynamic compromise. A decision for PTCA can™t be made unless angiography provides specific indications for this procedure.

98. Infective endocarditis prophylaxis and repeated regular follow-ups are recommended for all patients of aortic stenosis even if they are asymptomatic.

99. Renal artery stenosis is a common cause of resistant HT in a patient with advanced atherosclerosis. Carefully auscultate the periumbilical area of such a patient to reveal continuous (or systolic with diastolic component) murmur characteristic of renal artery stenosis.

100. Atrial myxomas can present with systemic features and findings similar to MS.

101. TEE (transesophageal echocardiography) or computed tomography are the Dx studies of choice for suspected aortic dissection.

102. EKG manifestations of digitalis toxicity: atrial tachycardia with AV block. (more specific).

103. Digoxin causes ST segment depression, T wave inversion, first degree AV block at therapeutic levels and they do not represent digitalis toxicity and therefore there is no need for discontinuation of the drug.

104. MVP is the MCC of isolated mitral regurgitation in North America.

105. Controlling the rhythm or rate in patients with prolonged tachysystolic atrial fibrillation usually improves the LV function significantly, sometimes even dramatically.

106. Presence of hypotension, pulsus paradoxus, and pulseless electrical activity in a patient with a recent acute MI should make you think of free ventricular wall rupture.

107. The Tx of verntricular fibrillation is STAT defibrillation with 200-360 joules. If defibrillation fails, lidocaine or amiodarone (DOC) can be loaded and the patient shocked again. Epinephrine can sensitize the heart and lower the threshold for conversion.

108. Premature atrial beats are benign and neither
Reply
#38
This is what Dr. Fischer believes is absolute high yield for seizures. Try to think of the answer before looking at it, its sth only way you remember it:

1 - Dx and Tx of Absent seizures -
2 - Pregnant c seizures tx -
3 - Adverse effect of phenytoin -
4 - Adverse effect of Lamotrigine -
5 - Driver c h/o seizures, next step -
6 - Diagnostic test to stop seizure meds -
7 - Must be seizure free for how long before stopping meds - 8 - Do NOT treat first time seizures unless:
9 - Another disease you where use Carbamazepine -
10 - Status Epilepticus Diagnostic tests (IN ORDER) -
11 - Status Epilepticus Tx (In order) -

Answers:
1 - kid staring in space, blinking eyes/lip smacking, 3 waves, tx c ethosuximide
2 - Carbamazepine (or MgSO4)
3 - gingival hyperplasia, teratogenicity
4 - Steven-Johnson syndrome
5 - advise to stop (thats all you do, do not report him)
6 - Sleep-deprived EEG
7 - >2 years
8 - 1 - family h/o seizures, 2 - status epilepticus, 3 - EEG abnormality
9 - Trigeminal Neurolgia
10 - 1st - Serum Sodium, 2nd Serum Glucose, 3rd low calcium, 4th low O2, 5th Toxicology (TCA, Cocain, Benzo, barbs), 6th CT head, 7th EEG (LAST RESORT)
11 - 1st benzo, 2nd Phenytoin, 3rd barbiturate, 4th General anasthesia (does not stop seizure, just stops the shaking)

Reply
#39
HY Notes: CK(Q-bank)
¢ Familial short stature: normal birth weight and length ô€ƒŽ age of 2-3 years growth decelerate and drops to 5th percentile ô€ƒŽ The onset and progression of puberty normal : Bone age is typically consistent with the chronologic age.
¢ Immunizations: Hep B vaccine- At birth, 1 month, and 6 months DTaP- At 2, 4, and 6 months, 15-18 months, and 4-6 years Td booster- 11-12 years, and then every 10 years Hib- At 2, 4, and 6 months, and 12-15 months
IPV- At 2 and 4 months, 6-18 months, and 4-6 years
MMR- At 12-15 months and 4-6 years
Varicella- 12 months
¢ Chemo-prophylaxis tuberculosis: The usual agent isoniazid. Indicated in the following groups:
1) tuberculin skin test has converted from negative to positive within the previous 2 years;
2) all small children (<4 years of age)exposed by known close contact to a person with untreated tuberculosis or who have a positive PPD; In young children, not to delay chemoprophylaxis until the PPD test becomes positive in child,
3) all HIV patients with positive PPD;
4) elderly patients with a definite conversion of PPD;
5) PPD positive persons with apical scars;
6) PPD positive persons with significant risk of recurrence due to diabetes mellitus, prolonged corticosteroid therapy, gastrectomy, end-stage renal disease, or gastric stapling.
¢ CHARGE syndrome: colobomas, heart defects, choanal atresia, retardation, genitourinary abnormalities/ cryptorchidism and ear anomalies,not genetically transmitted and is not associated with a teratogenic effect of any substance.
¢ Amount of Factor VIII to hemophiliac A patients: weight in pounds x 20 x desired plasma level in units. (Not applicable in hemophiliac B patients because Factor IX tends to bind to the endothelium of the vessel walls.Monitor the patient for cessation of bleeding or check clotting times before assuming that a calculated dose of Factor IX had its intended effect)
¢ Ataxia-telangiectasia: autosomal recessive disease.Ataxia,Choreoathetoid movements, slurred speech, ophthalmoplegia, and progressive mental retardation,Telangiectasias.Recurrent sinopulmonary infections. lack of IgA and IgE, cutaneous anergy and a progressive cellular immune defect. predisposition for certain cancers (leukemias, brain cancer, and gastric cancer). Most of these patients die of their neurologic deterioration by age 30.
¢ Irreversible muscle weakness in Duchenne muscular dystrophy seen after ventilator put for respiratory distress due to pneumonia etc ô€ƒŽhypoventilation on weaning ô€ƒŽtracheostomy will reduce the dead space and airway resistance.
¢ Infants usually double their birth weight by 6 months
Infants are triple their birth weight by the age of 12 months. Infants usually quadruple their birth weight by 24 months. Infants usually double their length by 4 years.
¢ Most patients with ALL are significantly granulocytopenic = like aplastic anemia associated with bone pain & fever = features like ecchymoses & infections
¢ Orthostatic proteinuria:very common ô€ƒŽprotein excretion rate is higher while the child is in an upright position ô€ƒŽobtain a "first morning" urine before the child has had much time in an upright position. DDx Minimal change ds = Edema, hypercholesterolemia & hypoproteinemia
¢ Neonate with pneumonia and h/o conjunctivitis on the fourth day of life. The leukocyte count is elevated at 15,000 with 40% eosinophils = suspect Chlamydia
¢ Corrossive ingestion: ingestion can cause severe esophageal necrosis of the liquefaction type. Full-thickness injury is common. In severe cases, it can cause esophageal perforation and mediastinitis. Acid ingestion causes coagulation necrosis and eschar formation. The eschar tends to protect the esophagus from full-thickness injury and corrosive perforation. Perform an esophagoscopy within 24 hours of exposure for the following patients following alkali ingestion:
1. Small children
2. Symptomatic older children and adults
3. Patients with abnormal mental status
Esophagoscopy should not be performed in patients with evidence of gastrointestinal perforation, significant airway edema, or necrosis and in those who are hemodynamically unstable.
KaplanQbankNotes:Neeraj 1
¢ Children with sickle cell disease are at risk of serious bacterial infection and sepsis because they have impaired splenic function. Intravenous ceftriaxone is the most commonly used antibiotic in a febrile child with sickle cell disease. It is effective against both S. pneumoniae and H. influenzae.
¢ Red urine = hematuria, hemoglobinuria, myoglobinuria, certain foods or medications (e.g., rifampin, nitrofurantoin, chloroquine, azo dyes, beets, and blackberries), and the presence of urates.
+ Strip-test for blood in a urinalysis = red blood cells, hemoglobin or myoglobin = perform microscopic analysis to DDx Raised serum creatinine phosphokinase = rhabdomyolysis = admit for aggressive IV hydration and treatment with sodium bicarbonate to alkalinize the urine to prevent precipitation of the myoglobin in the renal tubules.
¢ Nightmares:frightening dreams that awaken the child from REM sleep recall details of the dream.
Nightterror: Stage 3 or 4 sleep,not recalled Rx simply let it go but prevent injury
¢ Functional abdominal pain is pain that lasts for more than 3 months and often interferes with normal activity. The pain is periumbilical and often hard to describe. The pain typically does not awaken patients from sleep or interfere with pleasant activities. The pain is real and is the result of the regulation of gastrointestinal motility in response to either psychological or physical stress.
¢ Fragile X male = Adv Cytogenetic testing in female siblings = Heterozygous females frequently have developmental and behavioral problems such as ADHD. They may also have borderline or mild mental retardation.
¢ Rx Acne vulgaris: retinoid (tretinoin, adapalene, and the new yeast-derived agent azaleic acid).S/E skin irritation & photosensitivity. Frequent face washing with strong soap will probably cause exacerbation of acne. Gentle face washing once or twice daily with mild soaps is recommended.
¢ Toxicities tricyclic antidepressant ingestion: 1. A prolongation of the QRS interval is highly predictive of both cardiac and CNS toxicities
2. Right deviation of the QRS axis (greater than 120 degrees) is very predictive of cardiac toxicity from tricyclics.
¢ Fat embolism brain = multimodal petechiae in the white matter represent the most common pathologic change. Corpus callosum and cerebral peduncles small petechiae = Diffuse axonal injury = coma a few hours to days after head trauma. Multiple cortical infarcts, usually of the hemorrhagic type = Septic embolism / white matter is spared.
¢ Prostatic abscess:Infecting organisms include aerobic gram-negative bacilli and Staphylococcus aureus.
¢ Hyper IgM immunodeficiency = increased susceptibility to major gram-positive pathogens and opportunistic infections (such as the patient's Pneumocystis infection). The biochemical basis defect in a receptor on the T cell membrane that helps to trigger B cell switching from IgM to IgA, IgG, and IgE.
¢ The tidal volume for a patient on respirator is generally estimated as 10 mL/kg of weight.
¢ Screening every 5 years, a random cholesterol level should be checked = > 240 mg/dL =fasting lipid profile
¢ Scaling skin on back and extensor surfaces of the extremities/flexor surfaces are uninvolved. Cracking of the skin is prominent on the palms and soles. + atopy = ichthyosis vulgaris, Rx minimizing bathing with use of soaps only in the intertriginous areas. Bathing limited to 10-minute periods (to hydrate the stratum corneum), followed by immediate application of an emollient such as petrolatum, can help to control the scaling. In addition, 50% propylene glycol in water under occlusion by thin plastic film or bags during the night is helpful in adults, but is not usually used in children.
¢ Nummular dermatitis: chronic inflammation / etiology unknown. Coin-shaped/discoid itchy patches of vesicles and papules (ooze serum and crust over)numerous on the extensor surfaces of the extremities and on the buttocks. Heal and then reappear at the same sites. Microscopically, localized spongiosis (corresponding to edema) of the epidermis, which may also contain minute fluid-filled holes that correspond to the tiny KaplanQbankNotes:Neeraj 2
vesicles seen clinically in early lesions. Treatment of these patients is problematic, and numerous regimens involving corticosteroids or antibiotics have been recommended, each of which appears to work with some but not all patients.
¢
¢ Euthyroid sick syndrome: seriously ill patient with low T4 /T3, but below normal, normal, or minimally elevated TSH and clinically do not have clinical hypothyroidism.DDx true hypothyroidism (significantly raised TSH).
¢ End-stage liver disease/Cirrhosis ô€ƒŽ renal vasoconstriction occurs worsening fatigue and confusion ô€ƒŽ distal convoluted tubule responds by conserving sodium ô€ƒŽ diminished urine output and low urinary sodium ô€ƒŽ Deranged BUN/creatinine ô€ƒŽ hepatorenal syndrome ô€ƒŽ Rx Liver transplantation ô€ƒŽ reverse this vasoconstriction and kidney function will return to normal.
¢ HIV encephalitis (AIDS dementia complex): most common CNS complication in AIDS = cognitive impairment, incontinence, impairment of motor skills, and confusion = sub acute inflammatory infiltration of the brain caused by direct spread of HIV = diagnosis of exclusion = MRI studies and CSF analysis are useful in excluding other CNS diseases.
¢ Patient with respiratory distress due to pneumonia ô€ƒŽ oxygen saturation is 80% on room air ô€ƒŽHypoxia puts at significant risk for delirium, cardiac arrhythmias, and cardiopulmonary arrest ô€ƒŽ Oxygen should first be administered noninvasively ô€ƒŽstarting with a non-rebreather face mask ô€ƒŽFailed ô€ƒŽ go for invasive methods like endotracheal intubation.
¢ Hospitalization in community-acquired pneumonia :Criteria for hospitalization: one of the following is needed:
1. respiratory rate > 30 breaths/min,
2. room air PaO2< 60 mm Hg,
3. O2 saturation less than 90% on room air,
4. or bilateral or multiple lobes involved
¢ The normal value for the anion gap is 12 ± 4 mEq/L
¢ Gaucher disease: deficiency of the enzyme glucocerebrosidase (Ashkenazi Jews ) ô€ƒŽprogressive accumulation of glucocerebroside within lysosomes of histiocytes ô€ƒŽ Gaucher cells, large histiocytes with their cytoplasm engorged with glycolipid = "crumpled tissue paper" appearance ô€ƒŽmost common adult variant (type I) =most severely affected organs are the bone marrow, liver, and spleen ô€ƒŽ Bone marrow involvement =progressive pancytopenia and bone fractures ô€ƒŽ Glucocerebrosidase levels in circulating leukocytes: diagnostic. In the US, the disease is Rx glucocerebrosidase named alglucerase(safe but expensive)
KaplanQbankNotes:Neeraj 3
¢ an erythematous, slightly raised, 2-cm patch of skin on sun-exposed area (Face mostly) ô€ƒŽ area has a rough, very adherent, yellow-brown scale ô€ƒŽ Actinic keratosis ô€ƒŽRx application of 5-fluorouracil cream twice daily x 4 one-week cycles alternated with no treatment weeks ô€ƒŽheal without scarring.
¢ Ventilator delivering high concentrations of oxygen ô€ƒŽ lead to irreversible pulmonary fibrosis ô€ƒŽUse positive-end expiratory pressure (PEEP) to prevent the development of oxygen toxicity ô€ƒŽBut PEEP does increase the risk of both barotrauma and hypotension by impairing right-sided heart filling.
¢ Secondary hyperparathyroidism developed in renal failure in an attempt to correct the hypocalcaemia ô€ƒŽ bone calcium deficits and pathologic bone lesions ô€ƒŽRx Calcium supplementation in Renal failure
¢ Atypical nevus(Dysplastic)Confusedize from 5 to 12 mm ô€ƒŽmost commonly on sun-exposed skin ô€ƒŽround in color, but some have subtly notched borders or are slightly asymmetrical ô€ƒŽ intermediate category between obviously benign nevi and malignant melanoma ô€ƒŽincreased rate of progression to melanomaô€ƒŽ Isolated dysplastic nevi are often excised to remove the melanoma riskô€ƒŽcases with large numbers ô€ƒŽ careful monitoring with serial photographs can identify any changing lesions which may be undergoing malignant transformation.
¢ Post-MI patients: Ejection fraction near or minimally subnormal Rx Beta blockers ô€ƒŽdecreasing both oxygen demand and the incidence of ventricular arrhythmia ô€ƒŽ Improved survival But if ejection fractions less than 40% ô€ƒŽ Rx Angiotensin-converting enzyme (ACE) inhibitors
¢ Tension headache: headache often triggered or worsened by stressful situations, anxiety and fatigue Rx NSAIDs
¢ Eosinophilic fasciitisrange peel skin on the anterior aspects of the extremities= scleroderma-like disorder involving the arms, legs, and sometimes face and trunk, but not usually the hands and feet ô€ƒŽ lead to eventual restriction of arm and leg motion related to inflammation and fibrosis of fascia ô€ƒŽBiopsy of the skin or fascia shows cellular infiltrates with histiocytes, plasma cells, lymphocytes and (in only some cases, despite the name) eosinophils ô€ƒŽ Rx high dose prednisone followed by tapering and maintenance for 2 to 5 years on low dose prednisone.
¢ Pulsus paradoxus: significant fall in systolic blood pressure with inspiration = severe asthmatic attack ô€ƒŽ Look for associated accessory muscles of respiration, i.e., the internal intercostal muscles and the sternocleidomastoid muscles.
¢ Traumatic hemolytic anemias: repeatedly compressed tiny blood vessels, causing fragmentation of some red cells (triangle and helmet shapes)ô€ƒŽ e.g. forced march DDx rhabdomyolysis: high creatine kinase
¢ Atrial fibrillation (in CHF) ô€ƒŽcauses cardiovascular embarrassment and pulmonary edema ô€ƒŽadequate management of atrial fibrillation is rate control ô€ƒŽ Rx Digoxin with or without a nodal agent such as a beta blocker.However,in sinus tachycardia seen with hyperthyroidism DOC is propanolol alone.
KaplanQbankNotes:Neeraj 4
¢ Bleeding in diverticular ô€ƒŽ usually due to the disruption of an often single arteriole or small artery in a diverticulum ô€ƒŽvessel can be sufficiently stretched by the diverticulum that it cannot undergo contraction.
¢ Sheehan syndrome: panhypopituitarism due to intrapartum necrosis of the pituitary ô€ƒŽinsulin challenge ô€ƒŽMeasuring plasma levels of growth hormone and cortisol and evaluating thyroid hormones and TSH levels ô€ƒŽRx hydrocortisone (IV not oral like prednisolone) and thyroid hormones.
¢ Blood supply from 1978 to 1985 was likely to be tainted with HIV positive blood ô€ƒŽpatients with a history of blood transfusions during these years even if currently asymptomatic, should be screened.
¢ Hyperkalemia ô€ƒŽFirst to give: IV calcium gluconate ô€ƒŽ to counteract the effect of the high potassium on the heart and muscle
¢ Hypercalcemia ô€ƒŽ Rx IV saline and furosemide ô€ƒŽrapid and safe way to lower serum.
¢ hemochromatosis = total body iron load >5 g; (hemosiderosis = milder iron overload)
¢ cirrhosis and portal hypertension ô€ƒŽ Rx Propanolol (to reduce his portal pressure) and Frusemide to relieve ascites ô€ƒŽ dehydration (dry mucous membranes) related to his diuretic ô€ƒŽ BUN elevated ô€ƒŽmay exacerbate hepatic encephalopathyô€ƒŽ but pulse does not demonstrate a reflex tachycardia because of the propranolol.
¢ In portal hypertension strictly restrict or entirely avoid any medications with a sedative effect, e.g., benzodiazepines as they may ppt an encephalopathy.
¢ Silent lymphocytic thyroiditis: common disorder of postpartum women,autoimmune reaction to the thyroid gland ô€ƒŽproduce transient hyperthyroidism (related to follicle destruction) followed by hypothyroidism (that may be either permanent or resolve within 1 year).
¢ Seborrhic Keratosis: This lesion is characterized by light brown to black papules or plaques with an adherent waxy, greasy scale. The "stuck-on" appearance is very characteristic. It is most often found on the face and trunk.
¢ The findings of a coagulopathy or of an encephalopathy confer the worst prognosis in patients with acute viral hepatitis.These findings, in fact; suggest the possibility of fulminant hepatic failure.
¢ IgA type heavy chain diseas:centered in the Middle East ,ages of 10-30ô€ƒŽpresent with abdominal mass and malabsorption (behaves like an abdominal lymphoma but not mailgnant)ô€ƒŽconfined to the gut and mesenteric lymph nodes ô€ƒŽmay represent an aberrant reaction to some sort of bacterial infection ô€ƒŽ Rx corticosteroids, cytotoxic drugs and broad-spectrum antibiotics.
KaplanQbankNotes:Neeraj 5
¢ Pemphigus vulgaris:uncommon autoimmune skin disorder characterized by blistering and erosions involving the mucous membranes and skin ô€ƒŽ autoimmune attack is on the junctions between epithelial cells in the epidermis ô€ƒŽ blisters occur high in ô€ƒŽthe epithelium and can rupture easily(Nikolsky's sign) ô€ƒŽ begin in the mouth ô€ƒŽ rapid rupture of the blisters may lead to the impression that the initial lesion is an ulcer rather than a blister ô€ƒŽ IgG deposition on epithelial cell surfaces ô€ƒŽlife-threatening as a result of fluid/electrolyte imbalance, secondary infection, or complications of the high-dose corticosteroid therapy.
¢ Marrow fibrosis suggests myelofibrosis. The marrow in aplastic anemia is fatty, rather than fibrotic
¢ Excess vitamin C supplementation can lead to uricosuria and the development of calcium oxalate stones.
¢ The perfusion territory of the anterior spinal artery includes the anterior horn cells and part of the pain and temperature pathways. Thrombosis of this artery causes flaccid paralysis, loss of bowel and bladder function and loss of pain and temperature sensation
¢ Infectious Mononucleosis: Ampicillin therapy leads to a maculopapular rash. An antibiotic should not be given.
¢ To rapidly assess for the possibility of antifreeze ingestion, the physician can evaluate the patient's urine under a Wood's lamp for fluorescence. Manufacturers of ethylene glycol-containing antifreezes typically add fluorescein to the mix, which will fluoresce under a Wood's lamp.
¢ Paroxysmal nocturnal hemoglobinuria(Marchiafava-Micheli syndrome):genetic defect in glycosyl-phosphatidyl-inositol "anchor ô€ƒŽ marked sensitivity of RBC to serum complement factor C3 ô€ƒŽ Hemolysis triggered by infection, iron use (prescribed to treat the anemia), vaccination, or menstruation ô€ƒŽpredisposed for thrombotic disease, including Budd-Chiari Syndrome ô€ƒŽHam test:classic but non-specific:acid incubation causes red cell lysisô€ƒŽflow cytometric analysis using CD 55 and CD 59 (most definitive).Prognosis good only a few require allogenic bone marrow transplant.
¢ AMI : Thrombolytic therapy is indicated in patients up to 75 years of age .Absolute contraindications include a bleeding diathesis, major surgery or trauma within 6 months, gastrointestinal bleeding, or the presence of aortic dissection or a known intracranial tumor.
¢ Whenever you see ˜Lewy™s Body™ think of relation with Parkinson™sô€ƒŽ dementia with visual hallucinations and extrapyramidal signs ô€ƒŽdementia with Lewy bodies ô€ƒŽ Like parkinsons it shows fluctuating clinical course with alternating periods of improvement and deterioration ô€ƒŽDDx Pick dementia =Personality changes & disinhibition,affecting younger than 65 ô€ƒŽstriking atrophy of the frontal and anterior temporal lobes.
¢ Suspected lung carcinoma ô€ƒŽhoarsenessô€ƒŽmetastatic disease to the recurrent laryngeal nerveô€ƒŽincurability by surgical means.
¢ Charcoal is DOC within 4 hrs of toxic ingestion in any case ô€ƒŽ also applicable to Acetominophen ô€ƒŽGive charcoal + N-acetylcysteine within 4hrs of ingestion ô€ƒŽIf >4 hrs but <10 hrs = N-acetylcysteine alone ô€ƒŽ continue oral treatment for 72 hours.
¢ Tangier disease:alpha-lipoprotein deficiency ô€ƒŽvery low high-density lipoprotein (HDL)ô€ƒŽrecurrent polyneuropathy, lymphadenopathy, and hepatosplenomegaly due to storage of cholesterol esters in KaplanQbankNotes:Neeraj 6
reticuloendothelial cells 􀃎orange-yellow tonsillar hyperplasia (due to the cholesterol ester deposits) is a distinctive clue
¢ HIV patient with bloody diarrhea + tensmus + urgency/fecal incontinence ô€ƒŽ Sigmoidoscopy showing proctosigmoiditis with deep ulcers ô€ƒŽ suspect CMV
¢ Obese patient? in postop state ô€ƒŽ developing respiratory distress ô€ƒŽ think about PE
¢ Nephritic syndrome with bland urine in drug users who "skin pop" their drugs and have recurrent infections ô€ƒŽ Enlarged kidneys ô€ƒŽ ?Amyloidosis DDx Heroin Nephritis=Small kidneys
¢ Idiopathic hypertrophic subaortic stenosis (Earlier name of Hypertrphic cardiomyopathy):frequent cause of syncope or near syncope in young patients ô€ƒŽcharacteristic murmur by its increase with the Valsalva maneuver (Any maneuver that decreases left ventricular size will increase the murmur because the obstructive component increases as the left ventricular cavity shrinks ô€ƒŽ systolic ejection murmur is diminished when the patient lies down (This increases cardiac size by increasing venous return and tends to diminish the intensity of the murmur.This increases the ventricular size and diminishes the murmur).Rx Beta-blockers, such as propanolol, help relax the left ventricular smooth muscle and reduce ventricular outflow obstruction.
¢ Mesalamine (active ingredient 5-aminosalicylic acid) rectal suspension, suppositories, delayed release oral tablets and controlled release oral capsules ô€ƒŽnot absorbed and acts topically ô€ƒŽ modulation of arachidonic acid metabolites, including prostaglandins, leukotrienes, and hydroxyeicosatetraenoic acids ô€ƒŽwell tolerated except sulfite sensitivity.
¢ Hypertensive emergency (Raised both systolic & diastolic BP)may lead to increased Intracranial tension(Blurry vision/Papilledema) subarachnoid hemorrhage and end-organ failure (Like renal manifesting as ˜cola™ colored urine) ô€ƒŽRx Control BP but the blood pressure should not be lowered too far ô€ƒŽ systolic pressure in the range of 160-170 mm Hg because some of the elevated pressure may represent a compensatory mechanism to maintain cerebral perfusion pressure in the face of increased intracranial pressure or cerebral arterial narrowing (May lead to watershed infarcts)ô€ƒŽIV nitroprusside is a good agent because it can be titrated with the blood pressure. If the pressure drops too low, the IV can be turned off.
¢ IgG subclass deficiency: minor forms of immunodeficiency disease ô€ƒŽdeficiency may involve either or both IgG2 and IgG3 with or without IgG4 deficiency ô€ƒŽ (IgG1 is the major form, and its deficiency leads to a deficiency of total IgG (by definition not considered a "subclass" deficiency)ô€ƒŽpotentially clinically important point that patients with IgG2 deficiency may also have IgA deficiency and may develop anaphylaxis if given IgA-containing blood products.
¢ Chronic "autoimmune" hepatitisô€ƒŽ young age & hypergammaglobulinemiaô€ƒŽbiopsy demonstrating portal inflammation with lobular damage resulting in bridging necrosis
1. Type I(classic type: most frequent):associated with antinuclear and anti-smooth muscle autoantibodies
2. Type II(more common in women of Western European descent):associated with autoantibodies to circulating liver-kidney microsomes. insidious + amenorrhea.
¢ Chronic hepatitis C ô€ƒŽ positive enzyme immunoassay test for HCV-antibodies(this test may be falsely positive in situations with hypergammaglobulinemia like seen in chronic autoimmune hepatitis)ô€ƒŽpositivity should be confirmed by a more specific RIBA.
¢ Ganglion cyst: degenerative nontumourous/cystic swellings with gelatinous material having high hyaluronic acid content the center over dorsal aspect of the wrists, usually near or attached to tendon sheaths and joint capsulesô€ƒŽ Common sites:65%scapholunate joint,volar aspect of the radius and the flexor tendon sheath. ô€ƒŽregress spontaneously or after needle aspiration of the contents. Recurrent ganglia or ganglia that are cosmetically unacceptable to the patient can be surgically excised, but may recur after excision.
¢ Strenuous exercise like swimmingô€ƒŽdevelopment of headache, dizziness, One sided arm clumsiness and leg weakness + loss of pain and temperature sensation on same side face and contra lateral body areas ô€ƒŽ No prior illness ô€ƒŽ ? lateral medullary syndrome ô€ƒŽinvestigate vertebral artery dissection.
KaplanQbankNotes:Neeraj 7
¢ Diagnosis of Myocardial infarction requires either characteristic ST segment elevations on the ECG OR elevations in serum markers for cardiac injury. ˜Myocardial ischemia™ may or may not always manifest as MI(ô€ƒŽAngina). Diabetics often have silent ischemia.
¢ Propranolol is considered to be relatively contraindicated in patients with peripheral claudication & asthma.
¢ HIV/ lymphomas/organ transplants. ô€ƒŽEvidence of destruction of myelin at multiple sites in the CNS ô€ƒŽPMLô€ƒŽJC virus(papovavirus)ô€ƒŽ involves Oligodendrocytes in active lesions contain characteristic intranuclear inclusions.
¢ In constrictive pericarditis (calcification of the anterior pericardium) a pericardial knock is heard 0. 06-0. 12 seconds after the aortic valve closes. This corresponds to the sudden cessation of ventricular filling.
¢ Hypertrophic heart disease is the best recognized cause of diastolic dysfunction
¢ ECG ô€ƒŽP waves preceding the QRS complex but no two P waves have the same morphology ô€ƒŽ ˜Multifocal atrial tachycardia™ô€ƒŽVariable P wave morphology and PR and RR intervals ô€ƒŽ associated with severe pulmonary diseaseô€ƒŽcontrol of this tachycardia comes with improved ventilation and oxygenation.
¢ HSV is thought to cause encephalitis following transport to the brain along the trigeminal nerve.
¢ HIV with pneumocystis showing respiratory distress oxygen saturation on room air is 71% ô€ƒŽRx intravenous trimethoprim-sulfamethoxazole and prednisone ô€ƒŽgiven in any patient showing significant respiratory distress or room air oxygen saturation of less than 75 % ô€ƒŽotherwise in HIV Steroids are only given if the question of adrenal insufficiency, a common complication of HIV, is suspected.
¢ HIV patient with watery diarrhea ô€ƒŽtypical small bowel-type diarrhea(weakness & weight loss)ô€ƒŽmost common etiology cryptosporidiosisô€ƒŽspores can be seen on the tips of the villi on biopsy.
¢ Hypotension always accompanied with Tachycardia but when it is associated with Bradycardia ô€ƒŽIt is vagal response ô€ƒŽRx Atropine.E.g: post AMI
¢ Amyotrophic lateral sclerosis is a progressive motor neuron disease that affects both upper and lower motor neurons. Patients present with a gradual onset of asymmetric weakness of the distal limb. Even in advanced disease, sensory and bladder function are preserved. On exam, there is hyperactivity of muscle stretch reflexes.
¢ If autonomic dysfunction is suspected in a diabetic patient, a useful maneuver on physical examination is to look for a decrease in heart rate in response to the Valsalva maneuver (forced expiratory effort against a closed airway).
¢ Meniere disease = tinnitus, vertigo, and progressive hearing lossô€ƒŽ thought to be related to a degeneration of the vestibular and cochlear hair cellsô€ƒŽRx includes bed rest, a low-salt diet, dimenhydrinate, cyclizine or meclizine.
¢ Heparin-induced thrombocytopenia (HIT) is the result of platelet aggregation (platelet count falls below 50,000/mm3) caused by heparin-induced antibodiesô€ƒŽlead to limb-threatening thromboses and constitutes a medical emergency (Arterial thrombosis is a manifestation of the HIT syndrome)ô€ƒŽ Rx discontinue heparin and use lepirudin.
¢ An atypical gastric upset, heartburn, indigestion may turn out to be a myocardial ischemia ++> Look for objective signs of sympathetic activation including sweating, anxiety, tachypnea, and tachycardia or light-headedness, dyspnea, orthopnea, cough, nausea and syncope. An ECG examination is imperative in any patient presenting with this symptomatology. A normal tracing is rare with acute MI.
¢ Cerebellar bleed Cerebral bleed
abrupt onset develops slowly
lucid patient early loss of consciousness
cerebellar tonsillar herniation No herniation
should be evacuated as soon as possible before coma ensues No such need
KaplanQbankNotes:Neeraj 8
¢ The most common cause of chronic AF is valvular disease, followed by congestive heart failure (CHF). The most common anatomic correlate seen in patients with AF (Framingham Heart Study) is an enlarged left atrium.
¢ Otosclerosis may progress more rapidly in pregnancyô€ƒŽRx hearing aids; severe cases may respond to removal of the stapes with implantation of a prosthesis
¢ Amylodosis: echocardiogram reveals ventricular walls with a "speckled patternô€ƒŽ restrictive cardiomyopathyô€ƒŽ ventricular filling is impaired and the cardiac silhouette may be mildly enlargedô€ƒŽECG nonspecific arrhythmiasô€ƒŽ Like all restrictive ds develops into diastolic dysfunction.
¢ The pulmonary artery catheter/Swan-Ganz catheter allows direct, simultaneous measurement of pressures in the right atrium, right ventricle, pulmonary artery, and the filling pressure ("wedge" pressure) of the left atriumô€ƒŽ elevated right-sided pressure and low wedge pressure ô€ƒŽ Rt ventricular infarct? Causing backing up of venous blood and decreased forward flow, producing a decrease in left ventricular filling, ô€ƒŽRx aggressive fluid administration.
¢ A communication between an arteriole and venule in the cecum is a description of a vascular ectasia, also known as an arteriovenous (AV) malformation. This a common cause of painless colonic bleeding in the elderly and may present with acute gastrointestinal bleeding,chronic gastrointestinal bleeding, or iron-deficiency anemia. These lesions may be difficult to demonstrate, as the bleeding may be intermittent or the colon may be so full of blood that the site of origin is obscured. Techniques used to demonstrate bleeding AV malformations include colonoscopy, intraoperative endoscopy, and visceral angiography. Treatment of these lesions is problematic because many patients will subsequently develop new or recurrent bleeding vessels.
¢ Having difficulty holding and using a writing instrument due to hand and forearm spasms ô€ƒŽno medications, dizziness or loss of consciousness or any history suggestive of a seizure ô€ƒŽ focal dystonia of unknown cause.
¢ Cardioversionô€ƒŽSynchronized electrical cardioversion is the process by which an abnormally fast heart rate or cardiac arrhythmia is terminated by the delivery of therapeutic dose of electrical current to the heart at a specific moment in the cardiac cycle as determined by a compute0r(Pharmacologic cardioversion uses medication) ô€ƒŽSynchronized electrical cardioversion is used to treat hemodynamically significant supraventricular (or narrow complex) tachycardias, including atrial fibrillation and atrial flutter. It is also used in the emergent treatment of wide complex tachycardias, including ventricular tachycardia, when a pulse is present. Pulseless ventricular tachycardia and ventricular fibrillation are treated with unsynchronized shocks referred to as defibrillation.
So,hemodynamically unstable(loss of consciousness)vetricular tachcardia 􀃎treat with asynchronous cardioversion.
¢ Hypertensionô€ƒŽeye tries to protect itself from the hypertension ô€ƒŽ arteriolar constrictionô€ƒŽthickening of the arteriolar walls (producing the broad light reflex)ô€ƒŽ arterioles squeeze down too hardô€ƒŽ superficial foci of retinal ischemia(cotton wool spots)ô€ƒŽ hemorrhage and deposits occur because of vessel damage with leakage of contents.Hypertensive retinopathy seen in chronic essential hypertension, malignant hypertension, and toxemia of pregnancy. Rx control of the hypertension. (Practically, progression can be stopped and the hemorrhages will resolve, but the vessel changes remain.)
¢ Nonproliferative diabetic retinopathy= hemorrhage and exudates in the retina + microaneurysms (visible as red dots).
¢ Proliferative diabetic retinopathy = nonproliferative diabetic retinopathy +neovascularization with vessel growth into the vitreous.
¢ Brain-death ô€ƒŽno electrical activity and no clinical evidence of brain function on physical examination (no response to pain, absent cranial nerve reflexes ,pupillary response:fixed pupils, oculocephalic reflex, corneal reflexes), absent response to the caloric reflex test and no spontaneous respirations. But patient without such criteria but with no purposeful activity one week after an anoxic brain injury bodes poorly for a meaningful neurological recovery!(Not a brain death but recovery not meaningful).
¢ Whenever you see a case with a link to water reservoir/source like cruise/hospital/air-conditioner system and patient developed mental changes + diarrhea + Pneumonia = DDx Legionnaires pneumonia Rx Erythromycin
KaplanQbankNotes:Neeraj 9
¢ Yersinia enterocolitica: Right lower quadrant pain mimicking appendicitis with symptoms of an acute ileitis and diarrhea (In appendicitis patient typically have difficulty moving their bowels). Acquired through the fecal-oral route and diagnosis is based on clinical suspicion and the finding of the organism in stool cultures.
¢ Campylobacter jejuni: acute onset of either watery or bloody diarrhea consistent with colitis, severe fecal urgency, nocturnal bloody diarrhea ô€ƒŽsigmoidoscopy reveals continuous, symmetric inflammation from the anal verge to the proximal sigmoid colon ô€ƒŽ diarrhea in all age groups/peak of incidence is in young childrenô€ƒŽFecoral spread
¢ Intentional inhalation of volatile hydrocarbons(model glue, correction fluid, spray paint and gasoline) in a large quantity = "quick drunk": resembles alcoholic intoxicationô€ƒŽvery high dose ataxia, hallucinations, and nystagmus ô€ƒŽEncephalopathy is the major chronic morbidity (Hydrocarbons are highly lipophilic leading to CNS damage) ô€ƒŽhigh level of suspicion is needed ô€ƒŽno drug screen test that can detect inhalant hydrocarbons.
¢ Rubella vaccine ô€ƒŽGap of 3 months to be pregnant(theoretical risk of exposure to the rubella virus through vaccination)ô€ƒŽBut if pregnancy occurs within 3 months after vaccinationô€ƒŽnot an indication or a reason to terminate the pregnancy(Simply woman should be counseled about the theoretical risk).
¢ Neonatal especially preterm steatorrhea due to smaller bile acid pool ô€ƒŽsubstitute medium-chain triglycerides (MCTs) in the formula for long-chain triglycerides (LCTs)ô€ƒŽMCTs do not require bile acids for absorption.
¢ Milestones:
1. 4-years 􀃎 draw a four-sided figure (i.e., a square), count to 4, identify four colors, say a four-to five-word sentence, and draw a picture of a person with at least four parts. (Easily remembered as 4-year-olds do things in 4's.)
2. 5 yrs 􀃎Drawing a triangle,count to 10, repeat a 10-syllable sentence, and draw a picture of a person with 8-10 parts
3. 6 yrs 􀃎Building a staircase with cubes,can perform simple addition and draw a person with 12-15 parts.
4. 7- yrs 􀃎ability to repeat five digits,can repeat three digits backward, draw a diamond shape, and draw a person with 18-22 parts.
¢ Mitral valve prolapse =An apical click followed by a late systolic murmur ô€ƒŽRx antibiotic before Dental procedure Mitral and tricuspid regurgitation = produce holosystolic murmurs with relatively uniform intensity. Mitral regurgitation is heard at the apex while tricuspid regurgitation is best heard along the lower left sternal border. Mitral stenosis= mid-diastolic murmur heard after an opening snap.
¢ nasal foreign body: frequent sneezing and obstruction(Often misinterpreted as a common cold or allergy) ô€ƒŽinfection develops ô€ƒŽresulting in a purulent and malodorous dischargeô€ƒŽunilateral involvement
¢ A saturation of 90% corresponds to an oxygen partial pressure of around 60 mm Hg =Hypoxia needs oxygen
¢ hepatitis A diagnosis is best made by determination of IgM levels against hepatitis A virusô€ƒŽacute infection(antibody peaks at 4-6 weeks and does not persist beyond 6 months)while IgG is produced in the primary infection, but for most viral infections, including hepatitis A, it persists for a life time.
¢ Attention deficit/hyperactivity disorder (ADHD) is characterized by impulsivity, hyperactivity, and inattention lasting at least 6 months and disorder must have started before age 7.
¢ Cardiac defects:
1. Hypoplastic Lt heart syndrome: underdevelopment of the left cardiac chambers(left atrium and ventricle often exhibit endocardial fibroelastosis), atresia or stenosis of the aortic and/or the mitral orifices, and hypoplasia of the aorta􀃎 No murmur, precordial hyperactivity(enlarged right ventricle is contracting against systemic pressure), loud second heart sound (because the pulmonary artery acts as the aorta by pumping blood to the systemic circulation through the ductus arteriosus). When the ductus closes, or when the pulmonary vasculature resistance falls, the flow to
KaplanQbankNotes:Neeraj 10
the systemic circulation will decrease, causing greatly diminished peripheral pulses. Management includes infusion of prostaglandin E1 and administration of room air while on a ventilator. Prostaglandin E1 may open the ductus arteriosus and restore systemic blood flow.
2. Total anomalous pulmonary venous return:pulmonary veins forming a confluence behind the left atrium, and draining into the right atrium 􀃎Complete mixing takes place in the right atrium 􀃎 right-to-left shunt through the foramen ovale to the left side of the heart􀃎 Often, no murmur is heard 􀃎chest roentgenogram often shows a normal heart size with pulmonary edema. If there is obstruction to pulmonary venous return, as is almost always present with veins draining inferior to the diaphragm, cyanosis can be very prominent.
3. Neonate becomes cyanotic then lost consciousness following feeding or when crying vigorously 􀃎 baby picked her up and held 􀃎infant regained 􀃎 hypercyanotic spells or "Tet spells"􀃎TOF
DDx Frequent cyanosis and difficult breathing following birth but improves while crying􀃎Choanal atresia Rx intubation via oropharynx
¢ Post infectious Bells palsy:Mostly EBV
¢ CHF in infant: respiratory distress, tachycardia and/or hyperdynamic precordium, and cardiac enlargement evidenced by echocardiogram. DDx left-to-right shunt (ASD, VSD, PDA, atrioventricular canal, or AV fistula), left-sided obstruction leading to myocardial dysfunction (severe coarctation or AS), or intrinsic myocardial dysfunction (myocarditis, cardiomyopathy, or infarct due to anomalous coronary artery). Rx IV furosemide provide quick symptomatic relief and improve respiratory distress.
¢ Rx streptococcal pharyngitis is oral penicillin V for 10 days. However, Benzathine penicillin G can be given as a single dose intramuscularly. It is a long-acting antibiotic and can complete the treatmentrevention of Rheumatic heart Ds in a noncompliant patient
¢ Patients with sickle cell disease are at risk for infection by Salmonella and other gram-negative bacteria increased risk for acute recurrent Salmonella osteomyelitis. However, even in patients with sickle cell disease, Staphylococcus aureus is still the most common pathogen for osteomyelitis.
¢ Children <3 years with harsh, barking cough(sounds like a seal),runny nose but no fever or drooling + hoarse with inspiratory stridor ô€ƒŽ Acute laryngotracheobronchitis (viral croup) DDx
1. Epiglossittis 􀃎fever, drooling, muffled voice, cyanosis, and soft stridor.
2. Laryngomalacia 􀃎persistent stridor that is first noted early in infancy
¢ Tanner stages:
1. Stage I (0-15yrs): preadolescent breast development with no pubic hair.
2. Stage II(8-15 yrs)breast budding or thelarche + small pubic hair near the labia + growth spurt
3. Stage III :more pubic hair
4. Stage IV (10-17yrs) breast and nipple enlargement with some contour separation of the areola is noted. Hairs adult quality but not distribution.
5. Stage V (12-18 yrs) complete breast enlargement with no contour separation of the areola. Pubic hair is of adult quality and distribution.
¢ Neonatal Group B Streptococcus (GBS): Early-onset / first week of life: involve the respiratory tract causing pneumonia .Late-onset (weeks 2 and 4): meningitis.
¢ Long Q-T syndrome (LQTS):50% familial: syncope and sudden death from torsades de pointes (TDP).
1. Romano-Ward syndrome has autosomal-dominant transmission;
2. Jervell-Lange-Nielsen syndrome has autosomal-recessive transmission.(High Mortality)
On ECG 􀃎 QT prolongation (QTc averages 0. 49 seconds).
Rx beta-blocker or implantable cardiac defibrillator in resistant cases.
¢ Child's recurrent leg painsô€ƒŽBilateral(Organic pain :Unilateral) ô€ƒŽoccurring soon after going to bedô€ƒŽrelieved from rubbing his legs and kneesô€ƒŽ no limp and is able to participate in sports activities ô€ƒŽ"growing" pains but unrelated to growth(affect 3-10 years).
¢ Metabolic alkalosis in cystic fibrosis due to excess sweating (summer)infants & dehydrationô€ƒŽ hyponatremic, hypochloremic, metabolic alkalosis
KaplanQbankNotes:Neeraj 11
¢ Breast-feeding jaundiceô€ƒŽdecreased intake and increased enterohepatic circulation.
¢ Any ingested toxic substance/overdose presented within 60 mintsô€ƒŽ Gastric lavage+ Add charcoal if possible
¢ Hypernatremic dehydration: hypotonic fluid loss (history of diarrhea or vomiting),inadequate supply of mother's milk or high concentration of sodium in mothers milk ô€ƒŽ irritable,lethargic infant and have a high-pitched cry ô€ƒŽComplication like intracranial hemorrhage
¢ Intussusception usually occurs within the 6 to 12-month-old age group
¢ Conscious Sedationô€ƒŽ minor procedures like suturing laceration ô€ƒŽ able to maintain airway patency, protective airway reflexes and responses to physical stimuliô€ƒŽUse short-acting or long-acting benzodiazepine (midazolam or diazepam, respectively) by the oral or rectal route for (Intravenous midazolam or diazepam can be used for procedures that produce more intense pain or discomfort, such as repair of complex lacerations, bone marrow aspiration, and reduction of fractures)
¢ "Do's" and "Don't's." during the seizure episodeô€ƒŽdon't Put any object into the patient's mouth,do Place the patient on the side,Put a pillow or other soft object under the patient's head,Loosen tight clothing around the neck,Remove sharp objects from the surroundings.After the seizures, caretakers or parents should remain with the patient until he/she is fully alert and allow him/her to go back to the usual activities.
¢ Child with ecchymosesô€ƒŽEliminate bleeding abnormalities by coagulation studies ô€ƒŽNormal studyô€ƒŽThink about Abuse!
¢ Triad of thrombocytopenia (hemorrhage may be the presenting complaint), eczema, and recurrent infections (often respiratory)ô€ƒŽ Wiskott-Aldrich syndrome(X-linked recessive)ô€ƒŽ defects in both T and B cell functionô€ƒŽvulnerable to pyogenic bacteria, viruses, fungi and Pneumocystis cariniiô€ƒŽoften died by age 15 and survivors have increased incidence of cancer (lymphoma /ALL) ô€ƒŽRx splenectomy, continuous antibiotic therapy, IV immunoglobulin, and bone marrow transplantation.
¢ Severe combined immunodeficiency (Autosomal recessive) ô€ƒŽ "bubble boy" disease because its victims are extremely vulnerable to infectious diseasesô€ƒŽadenosine deaminase deficiency (purine salvage enzyme and deficiency leading to turn off DNA synthesis)ô€ƒŽLow T/B cells counts.
¢ X linked SCIDô€ƒŽmutations in gene encoding the common gamma chain (γc),shared by the receptors for multiple interleukinsô€ƒŽ affects development and differentiation of T and B cellsô€ƒŽAll males with the defective gene will have SCIDô€ƒŽ Females are carriers.
¢ Bruton agammaglobulinemiaô€ƒŽCellular immunity is intactô€ƒŽ mature B-lymphocytes do not formô€ƒŽgerminal centers are absent in lymph nodes ô€ƒŽinfections after about six months (maternal antibodies have decreased)ô€ƒŽrecurrent pyogenic infections, particularly of the lungs, sinuses, and bones.
¢ Common variable immunodeficiency syndromeô€ƒŽClinically same as Bruton™s but onset in late adolescence/young adulthoodô€ƒŽhypogammaglobulinemia with markedly decreased IgMô€ƒŽfailure of B-lymphocytes to differentiate into plasma cells ô€ƒŽpatients have an increased risk for B-cell lymphomas, gastric carcinoma, and skin cancer
¢ Meconium =fetal stool(desquamated cells from the gastrointestinal tract admixed with enough bile to give the soft stool a greenish color)ô€ƒŽdistressed fetus will pass meconium into the amniotic fluid and then may aspirate itô€ƒŽIn maternal preeclampsia, hypertension, or postmaturityô€ƒŽaspirated meconium is very irritating to the lungs and causes a chemical pneumonitisô€ƒŽRx prompt suction of the nasopharynx and mouth
¢ Kawasaki diseaseConfusedystemic vasculitis of unknown originô€ƒŽmucocutaneous lymph node syndromeô€ƒŽClinical and echocardiographic features remain the basis for diagnosisô€ƒŽAn (unidentified) infectious origin and a T-cell immune activation ô€ƒŽFever, bilateral non-exudative conjunctivitis, mucous membrane changes (injected pharynx, cracked lips, or strawberry tongue), extremity changes (edema, desquamation, erythema, or rash), and cervical adenopathy ô€ƒŽmyocarditis, valvular insufficiency, arrhythmias, pericardial effusion, and congestive heart failureô€ƒŽLeukocytosis and an elevated C-reactive protein are associated with the development of coronary artery aneurysmsô€ƒŽRx aspirin ,IV immune globulin & Corticosteroids(Controversial:Reduces coronary artery aneurysm?).
KaplanQbankNotes:Neeraj 12
¢ The most commonly abused drug by pregnant mothers is cocaineô€ƒŽInfants small for gestational age (SGA) and sometimes,microcephaly and neurodevelopmental abnormalitiesô€ƒŽirritable and cries in high-pitchô€ƒŽ. increased risk of sudden infant death syndrome & Periventricular leukomalacia.
¢ Interesting facts that pancreatitis may show normal amylase(Rely more on lipase)
¢ Interesting facts that Gullain Barre synd may rarely have sensory involvement
¢ Listeriosis acquired by mothers exposed to unpasteurized dairy products or raw vegetables exposed to cattle or sheep manureô€ƒŽamnionitisô€ƒŽabortion, stillbirth or neonatal sepsisô€ƒŽbrown, murky amniotic fluid ô€ƒŽor Disseminated disease in the fetus can cause granuloma formation (with associated tissue destruction) in many tissues, including liver, adrenal glands, lymphatic tissue, lungs, and brain(granulomatosis infantiseptica)
¢ Prenatal toxoplasmosisô€ƒŽRx Treatment is with combination antibiotic therapy, including pyrimethamine, sulfadiazine, and leucovorin.
¢ Bupropion ô€ƒŽ antidepressant with both dopaminergic and noradrenergic propertiesô€ƒŽimprove depression and cognitive functioning (term cognition is used in several loosely related ways to refer to a faculty for the human-like processing of information, applying knowledge and changing preferences)ô€ƒŽalso used to reduce Nicotine craving/dependence ô€ƒŽAny drug with anticholinergic properties might impair cognitive function(even SSRI)
¢ Propranolol known to occasionally lead to depressive symptoms
¢ Loosening of associationsô€ƒŽideas are disconnected and seem to jump from one topic to an unconnected topic. Clang associationsô€ƒŽWords that rhyme are frequently associated. Concrete thinking ô€ƒŽpoor ability to think in abstract terms despite normal intelligence. Tangential thoughtsô€ƒŽthoughts that go off on a tangent, interviewer is commonly left with the sense that a question to the patient elicited a long string of thoughts that ended up having nothing to do with the original question. Thought blocking ô€ƒŽoccurs when thoughts and speech halt, often in mid-sentence, as if forgotten. The thought may be picked up later, after a period of apparent confusion.
¢ Cimetidine known to cause psychiatric effects like delusions and psychosisô€ƒŽrelated to the effects of cimetidine on the H-2 histamine receptor in the brain.
¢ Electroconvulsive therapy (ECT) has been safely used in pregnancy. In case of psychotic depression with increased risk for suicide, the situation requires expeditious treatment to protect the mother and fetus, and ECT is the treatment of choice.
¢ Post-traumatic stress disorderô€ƒŽRx Sertaline
¢ Maternity blues is a normal state of sadness, dysphoria, frequent tearfulness, and dependence that about 20% to 40% of women experience in the postpartum period. It is thought to be derived from rapid changes in women's hormonal levels and the stress of childbirth associated with maternity
¢ Behavioral therapy is the most frequently used treatment in children with enuresis. Dry nights are recorded on a calendar and rewarded with a star as a gift.
¢ Agoraphobia(fear of open spaces or of the marketplace)is a fear of panic attacks in situations from which it would be difficult to gracefully remove oneself. Behavioral therapy is used to encourage patients to modify their activities.
¢ Generally conventional antipsychotic + prophylactic anticholinergic agent (benztropine, diphenhydramine, or trihexyphenidyl).is given togetherô€ƒŽsudden ceassation of anticholinergicô€ƒŽPpt Extrapyramidal signs like dystonic reaction/torticollisô€ƒŽRx IM administration of an anticholinergic agent
¢ Although risperidone is an atypical antipsychotic, it is like conventional antipsychotics in its ability to cause significant elevations in plasma prolactin levels.
KaplanQbankNotes:Neeraj 13
¢ Psychomotor agitation is a series of unintentional and purposeless motions that stem from mental tension of an individual. This includes pacing around a room, wringing one's hands, pulling off clothing and putting it back on and other similar actions. In more severe cases, the motions may become harmful to the individual, such as ripping, tearing or chewing at the skin around one's fingernails to the point of bleeding DDx Psychomotor agitation in major depressive disorder > bipolar depression
¢ Catatonia include stupor, negativism(motiveless resistance to all attempts to be moved or to all instructions), rigidity, posturing, mutism, stereotypies, mannerisms, waxy flexibility, and catatonic excitement. Catatonia may be associated with schizophrenia (catatonic type), mood disorders (with catatonic features), or general medical condition or Extrapyramidal side effect of drug .
¢ Nihilistic delusions are false feelings that the self or others do not exist or are destroyed. It is typical for depression with psychotic features. At its extreme, it is called Cotard's syndrome.
¢ Prescribing antidepressants in bipolar depression may un-mask underlying mania ô€ƒŽStart mood stabilizer before beginning treatment with an antidepressant
¢ Panic disorder Rx benzodiazepine like clonazepam or use of cognitive behavior therapy, which incorporates exposing the patient to disturbing stimuli in an attempt to develop coping mechanisms in response to the stimuli.
¢ Adjustment disorder is exemplified by a set of behavioral or emotional symptoms developing as a response to an identifiable stressor within 3 months after exposure to the stressor. The symptoms are excessive compared with what one would expect from the exposure, and they cause marked impairment in social functioning.
¢ Any event that affects the vasculature, such as a myocardial infarction or a cerebrovascular accident (CVA), has been shown to increase the risk of major depressive disorder in the months following such an event. Post-stroke mania is a rare phenomenon usually seen in infarctions of right frontal lobe and sometimes other parts of the right hemisphere.
¢ Pemoline is a sympathomimetic agent approved for the treatment of attention deficit/hyperactivity disorder. Given the rate of reported cases of hepatic failure. Dextroamphetamine is approved for the treatment of attention deficit/hyperactivity disorder. Its main adverse reactions involve the cardiovascular system, the CNS, the gastrointestinal system, and the endocrine system.
¢ Prevalence of ADHD is estimated to decline by 50 % every 5 years until the mid-20's i.e. older the children get without symptoms of ADHD, the less likely they have the disorder. (Generally manifest before 6-7yrs)
¢ Mini-Mental Status Examination (MMSE) is a brief instrument designed to grossly assess cognitive functioning. It assesses orientation, memory, calculation, reading and writing capacity, visuospatial ability, and language. The maximum score is 30. can be re-administered periodically to follow progression of dementia.
¢ prochlorperazine and haloperidol administrationô€ƒŽ feeling of restlessness & agitation(akathisia)ô€ƒŽRx lorazepam given IV during administration of the neuroleptic drug.
¢ The most serious side effect of lamotrigine(adjunct in the treatment of refractory seizures and bipolar disorder) is rash (skin changes that looked like burns.)ô€ƒŽStevens-Johnson syndrome
¢ Clozapine is an atypical agent more effective in resistant schizophrenia. It exerts an antagonistic effect on D1 and D4 receptorsô€ƒŽS/E agranulocytosisô€ƒŽrequires regular blood count monitoring
KaplanQbankNotes:Neeraj 14
¢
¢
KaplanQbankNotes:Neeraj 15
¢ Thyroid nodule? Benign or malignantô€ƒŽnodule large enough to cause airway or digestive tract obstructionô€ƒŽ emergent thyroid surgery without further evaluation.
Step-1:TSH􀃎Decreased =hyperactive nodule=typically benignA
Step-2:TSH􀃎Normal or elevated TSH = non-functioning or normally functioning nodule.
Step-3:Imaging and/or a biopsy to obtain a tissue diagnosis􀃎ultrasound􀃎most useful diagnostic tool fine needle aspirate (FNA)􀃎result from an FNA
1. First, the sample can represent benign lesions= no further treatment (unless the lesion continues to grow causing obstruction to breathing or eating, or causing unacceptable cosmetic disfigurement in the neck).
2. Second, the specimen can be a follicular lesion = treated with a surgical removal of part or all of the thyroid gland to determine whether it is a benign or malignant type.
3. Third, the biopsy can show a clearly malignant lesion = treated with surgical removal of the thyroid gland.
4. Finally, the sample can be nondiagnostic = requires a repeat FNA. However, after three nondiagnostic biopsies, a surgical removal of the affected thyroid lobe is usually recommended.
¢ Approach towards Breast Lump:
1. Nipple discharge: Pathologic discharges are spontaneous, bloody or associated with a mass. These discharges are usually unilateral. The most common cause of pathologic nipple discharge is intraductal papilloma, followed by duct ectasia.10 If a palpable mass is present in association with a discharge, the likelihood of cancer is greatly increased.
2. Dominant breast massDx macrocyst (clinically evident cyst), fibroadenoma, prominent areas of fibrocystic change, fat necrosis and cancer. KaplanQbankNotes:Neeraj 16
(a)Solid Masses in Women Less Than 40 Years of Age
1. Physical Examination􀃎 No evidence of mass 􀃎reassured and instructed in breast self-examination.
2. Physical Examination􀃎 physical finding is uncertain 􀃎 directed ultrasound examination􀃎not demonstrate a mass􀃎Repeat physical examination􀃎?mammogram (35 to 40 years not in younger)
3. Physical Examination􀃎 dominant mass (? suspicious mass is solitary, discrete, hard and often, adherent to adjacent tissue)􀃎 mammography􀃎pathologic diagnosis.
4. Physical Examination􀃎 dominant mass (? Not a suspicious mass or clinically benign)􀃎 discus options of surgical excision or follow-up with the patient􀃎patient desires surgical excision􀃎no additional testing is done􀃎patient opts for further work-up􀃎an ultrasound examination and fine-needle aspiration are performed to confirm that the mass is benign. "Triple test" (clinical examination, ultrasonography [or mammography] and fine-needle aspiration).
¢ The size of the lesion must be measured with a ruler at presentation and on subsequent visits to allow an accurate assessment of size over time.
¢ patient is examined every three or four months for one year to ensure stability of the mass.
(B)Solid Masses in Women More Than 40 Years of Age.
1. Abnormalities detected on physical examination in older women should be regarded as possible cancers until they are documented to be benign.
2. mammography is a standard part of the evaluation of a solid breast mass.
3. In the presence of a dominant breast mass, a normal mammogram should never be considered proof of the absence of breast cancer.
¢ Premenstrual dysphoric disorder: constellation of physical and emotional symptoms occurring during the late luteal phase of the menstrual cycleô€ƒŽsymptoms must be present during most cycles in the past year and during at least two subsequent cyclesô€ƒŽat least five of the following symptoms: depressed mood, marked anxiety, affective lability, decreased interest, decreased energy, sleep disturbance, craving food, feeling overwhelmed, and difficulties concentratingô€ƒŽinterfere with social or occupational functioning and are not due to other psychiatric or medical disorders.
¢ Exposure therapyô€ƒŽa type of behavior therapyô€ƒŽmost commonly used treatment of specific phobiaô€ƒŽ desensitizes the patient by a gradual exposure to the phobic stimulusô€ƒŽRelaxation and breathing control are important parts of the treatment.
¢ Alprazolam belongs to the group of short-acting benzodiazepines. Even though it has a short half-life, it can produce confusion, disinhibition, and amnestic problems like blackouts in the elderly population. The risk is increased if it is combined with CYP3A inhibitors.
¢ In pseudodementia of depression, the patient often tends to emphasize disability related to memory loss much more than patients experiencing true dementia.
¢ Olanzapine is an atypical antipsychotic indicated for the management of psychotic disorders. It is said to be associated with weight gain.
¢ HIV dementia is characterized by affective, cognitive, behavioral, and motor symptoms and signs. It presents as a subcortical process and is most likely to occur in patients with a CD4 count below 200/mm3. It usually has a slow onset, and, after a period of stability, there can be a precipitous decline. The diagnosis is made when other causes of delirium are excluded; the disease may present with psychosis within HIV dementia. The symptoms are controlled with low doses of neuroleptics.
¢ Circumstantiality refers to speech that is delayed from reaching the point, characterized by overinclusion of details.DDxTangentiality : patient never gets to the desired goal from the starting point of discussion.
¢ Patients with disorganized type schizophrenia are likely to exhibit disorganized speech, disorganized behavior, and flat or inappropriate affect. Examples of disorganized thoughts and speech include: loosening
KaplanQbankNotes:Neeraj 17
of associations or derailment, flight of ideas, tangentiality, circumstantiality, word salad, neologisms, and clang associations.
¢ The newer atypical antipsychotics have minimal, if any effect on plasma prolactin concentrations, except for risperidone, which is associated with elevated prolactin.
¢ A Papanicolaou smear should ideally be a sampling of the transformation zone(adequate sample should show endocervical cells)ô€ƒŽendocervical cells not present?whether the transformation zone was fully sampledô€ƒŽRepeat the sample if high risk individual ô€ƒŽor after 1 yr if with no high risk and had all earlier pap normalô€ƒŽEvery woman should have yearly pap smear after first intercourse or 18yrs whatever comes earlier
¢ Rarely struma ovarii is a cause of hyperthyroidism and patients with this manifestation may have symptoms of hyperthyroidism, as well as elevated levels of thyroid hormones and decreased levels of thyroid stimulating hormone (TSH).
¢ There is no evidence that breast-feeding increases HCV transmission to the baby.
¢ Postpartum endometritis ô€ƒŽCesarean section is the major risk factorô€ƒŽOrganisms ascending from the girl thingy and causing a polymicrobial infection of the endometrium ô€ƒŽ fever and chills, lower abdominal pain, a foul-smelling vaginal discharge and malaiseô€ƒŽ abdominal tenderness, and uterine tendernessô€ƒŽRx clindamycin and gentamicin ( prophylactic antibiotics are recommended in all cases of nonscheduled cesarean delivery i.e. a cesarean delivery that is not anticipated like with membranes broke but non-progressing/Breech etc and are given before or after the umbilical cord is clamped)
¢ Discharge criteria ô€ƒŽpatient should be alert, able to ambulate (if this was her preoperative level of function), able to tolerate adequate oral intake(Patient on IV intake should not be discharged), have stable vital signs, and have satisfactory bowel and urinary tract function.
¢ Chorioamnionitis (can develop at any time before and during delivery)ô€ƒŽfever and uterine tendernessô€ƒŽRx ampicillin or penicillin with gentamicin.
¢ Asymptomatic bacteriuria in pregnant womenô€ƒŽassociation with preterm delivery/low birth weight ô€ƒŽRx trimethoprim-sulfamethoxazole, nitrofurantoin, and cephalexinô€ƒŽfollow-up urine culture after 10 days of completing the medicationô€ƒŽtest-of-cure.
¢ Gestational diabetesô€ƒŽusually diagnosed by means of oral glucose tolerance testingô€ƒŽPatients with gestational diabetes and normal fasting glucoseô€ƒŽtwo major risksô€ƒŽfetal macrosomia & eventual development of overt diabetes(within the next 20 years)ô€ƒŽwith gestational diabetes and abnormal fasting glucose ô€ƒŽincreased risk of stillbirth.
¢ Significant mitral stenosis during pregnancy should be monitored invasively using a Swan-Ganz catheterô€ƒŽ second stage of labor be shortened using forceps or vacuum to prevent excess maternal Valsalva efforts and maternal tachycardia.
¢ HIV Positive mother ô€ƒŽcombination of ZDV therapy + cesarean delivery decreases the risk of transmission to approximately 2% ô€ƒŽAmniocentesis should be avoided, if possible, in the HIV-positive woman.
1. 2% vertical transmission with ZDV + Scheduled CS prior to the onset of labor or rupture of membranes
2. 8% antiretroviral therapy in mother + Infant
3. 25% when none used
¢ Constitutional delay is normal pubertal progression at a delayed rate or onset. The average age at m
Reply
#40
bump for newcomers
G.L
Reply
« Next Oldest | Next Newest »


Forum Jump: